SlideShare uma empresa Scribd logo
1 de 90
Cases in cardiology
DR.MAGDI AWAD SASI
CCU
7TH
OCTOPER HOSPITAL
BENGHAZI
LMB—BENGHAZI UNIVERSITY
A 55-year-old man with a history of diabetes,
hypertension, hyperlipidemia, and current smoking
presents with a 2-month history of progressively
worsening chest pain on exertion. Which of the
following diagnostic procedures is most appropriate
A. Exercise ECG testing
B. Adenosine myocardial perfusion scintigraphy
C. Dobutamine stress echocardiogram
D. PET scan
E. Cardiac catheterization
 E
A 35 year old woman with a history of rheumatic fever in
childhood had had episodes of congestive heart
failure for several years. During the last of her
hospital admissions, she developed massive
pulmonary thrombo-embolism and died. Autopsy
examination showed that her lungs were firm and
brown, and microscopically had widened, edematous
and fibrosed alveolar septa, aggregates of heart
failure cells in the alveoli, thickened small muscular
arteries, and atherosclerosis in the larger pulmonary
arteries. An acquired valvular disease was found in
the heart. Which one of the following was it most
likely to be
A. MITRAL STENOSIS.
B. TRICUSPID STENOSIS.
C. AORTIC STENOSIS.
D. AORTIC REGURGITATION.
E. MITRAL REGURGITATION
The Correct answer is A
rheumatic heart disease (isolated mitral stenosis in
25% of all cases of rheumatic h.d., mixed mitral
stenosis and regurgitation in 40 % of all cases of
rheumatic h.d.,( two third of patients – women(
IN ABOUT 50% OF PATIENTS WITH MITRAL STENOSIS
THE RHEUMATIC FEVER MAY BE UNRECOGNISED
VIRAL CARDITIS - VERY RARE CAUSE OF MITRAL
STENOSIS
TWO THIRDS OF CASES OCCUR IN WOMEN
Pathophysiology
-the mitral valve orifice
-NORMALLY ABOUT 5 CM2 (IN DIASTOLE)
- MODERATE STENOSIS (APPROX 2CM2) - PATIENTS REMAIN
ASYMPTOMATIC
- 1CM OR LESS IN SEVERE MITRAL STENOSIS
-in mitral stenosis cardiac output may be maintained by rise pressure in LA,
pulmonary venous and capillaries - it results in the loss of lung
compliance and the development of exertional dyspnoea
-MILD STENOSIS - ATRIAL PRESSURE RISES ONLY ON EXERCISE
-SEVERE STENOSIS - RAISED ATRIAL PRESSURE IS REQIURED TO
MAINTAIN CO EVEN AT REST
-THE RAISED ATRIAL PRESSURE - ENLARGED DIAMETERS OF LA (LA
DILATATION IN 80% OS CASES) - ATRIAL FIBRILLATION (SOMETIMES
THE FIRST SIGN OF CLINICAL DETERIORATION) - LOSS OF ATRIAL
CONTRACTION CONTRICUTE TO DIMINISHED VENTRICULAR FILLING
- RISK OF ATRIAL THROMBOSIS AND SYSTEMIC THROMBOEMBOLISM
)PARTICULARY THOSE WITH AF(
-SUDDEN ONSET OF ATRIAL FIBRILLATION - SUDDEN INCREASE IN
PULMONARY VENOUS PRESSURE - SOMETIMES PULMONARY OEDEMA
- THE LONG-TERMED MITRAL STENOSIS AND MORE GRADUAL RISE IN
PRESSURE - THE INCREASED PULMONARY VASCULAR RESISTANCE,
WHICH PROTECTS AGAINST PULMONARY OEDEMA
- RIGHT VENTRICULAR HYPERTROPHY AND PULMONARY HYPERTENSION
(RESULTS FROM : PASSIVE BACK-PRESSURE, ARTERIORAL
CONSTRICTION, OBLITERATIVE CHANGES IN PULMONARY VESSELS
SYMPTOMS
)most common in the third or fourth decade of life(
EXERTIONAL DYSPNOEA, NOCTURNAL DYSPNOEA,
COUGH, HEMOPTYSIS
LEG, ANKLE OEDEMA, ABDOMINAL SWELLING (RIGHT
VENTRICULAR FAILURE(
SYMPTOMS OF ACUTE PULMONARY EDEMA (AF,
PREGNANCY(
SYMPTOMS SECONDARY TO ARTERIAL (RARE VENOUS)
EMBOLI
SYMPTOMS OF DIMINISHED CO (FATIGUE, TIREDNESS(
Asymptomatic mitral stenosis - the physical signs of
mitral stenosis are found before symptoms develop
SIGNS
ATRIAL FIBRILLATION
MITRAL FACIES
AUSCULTATION
LOUD FIRST HEARD SOUND (THE STENOTIC VALVE PROLONGS ATRIAL
EMPTYING, LEAFLETS REMAIN OPEN AT THE ONSET OF DIASTOLE AND
ARE CLOSED SUDDENLY(
OPENING SNAP (THE OPENING SNAP GETS CLOSER TO THE SECOND
SOUND AS THE STENOSIS BECOMES MORE SEVERE, MAY BE
INAUDIBLE IF THE VALVE IS HEAVILY CALCIFIED(
MID-DIASTOLIC MURMUR (THE TURBULENT FLOW CAUSES THE LOW
PITCHED DIASTOLIC MURMUR, AND OFTEN A THRILL; MURMUR IS
ACCENTUATED BY EXERCISE AND DURING ATRIAL SYSTOLE-
PRESYSTOLIC ACCENTUATION; IT IS HEARD BEST AT THE CARDIAC
APEX WITH THE PATIENTS IN THE LEFT LATERAL DECUBITUS
POSITION(
SIGNS OF RAISED PULMONARY CAPILLARY PRESSURE
(CREPITATIONS, PULMONARY EDEMA, EFFUSIONS(
SIGNS OF PULMONARY HYPERTENSION (ABNORMAL PULSATION FELT
TO THE LEFT OF THE STERNUM, DUE TO EITHER TO RIGHT
VENTRICULAR HYPERTROPHY OR TO FORWARD DISPLACEMENT OF
THE HEART BY DILATED LA; LOUD THE PULMONARY COMPONENT OF
THE SECOND HEART SOUND; TRICUSPID REGURGITATION
SECONDARY TO RIGHT VENTRICULAR DILATATION CAUSES THE
(SYSTOLIC MURMUR AND SYSTOLIC WAVES IN THE VENOUS PULSE(
INVESTIGATIONS
INVESTIGATIONS
THE ECG :
THE LEFT ATRIAL HYPERTROPHY - P MITRALE (THE BIFID P WAVES)
OR ATRIAL FIBRILLATION
RIGHT VENTRICULAR HYPERTROPHY (THE EARLIEST SIGN -
REDUCTION IN THE SIZE OF THE USUAL QS COMPLEX IN THE
LEAD V1(
CXR
ENLARGED LEFT ATRIUM
SIGNS OF PULMONARY VENOUS HYPERTENSION (ENLARGEMENT OF
THE UPPER PULMONARY VEINS, HORIZONTAL LINEAR SHADOWS
IN THE COSTOPHRENIC ANGLES(
ECHOCARDIOGRAPHY: THICKENED IMMOBILE CUSPS, ENLARGED
SIZE OF THE LEFT ATRIUM, REDUCED VALVE AREA, REDUCED
RATE OF DIASTOLIC FILLING
CARDIAC CATHETERISATION: PRESSURE GRADIENT BETWEEN LA
AND LV
A 45 year old man presented with gradually increasing
shortness of breath and episodes of paroxysmal nocturnal
dyspnea. His blood pressure was 140/40 mm of Hg, the
pulse was bounding, and a blowing diastolic murmur was
heard in the right upper parasternal region. Chest X-ray
showed marked widening of the mediastinum due to
dilatation of the ascending aorta. The most likely
diagnosis is
a. Pneumococcal endocarditis of the aorta valve.
b. Atherosclerotic aneurysm of the ascending aorta.
c. Syphiltic aortitis.
d. Mitral stenosis.
e. Calcific aortic stenosis
The correct answer is C
Pathological consequences of syphilitic aortitis
The aortitis is mostly inflammation in the media and adventitia,
induced scarring causes an intimal wrinkling in the aortic
wall
Tree bark appearance, which is chx of syphillis
Inflammation and fibrosis in the aorta, just above the aortic
valve cusp is very problematic
Can narrow the coronary arteries, producing severe ischemia
Aortic valve dilates à commisures separate from each other à
when the valves close, there are gaps at the periphery à
aortic regurgitation, which can be severe
Many cases die from this, rather than from the ruptured
aneurysm
65year old man had transfemoral catheterization for coronary
angiography which revealed the presence of severe
atherosclerotic narrowing of all major coronary arteries.
Within hours of the procedure, he developed severe left lower
quadrant abdominal pain. which was later associated with
rebound tenderness, livido reticularis (erythematous skin rash)
of the abdominal skin, increasing hypertension, cold blue toes
and progressively worsening renal failure.
Laparotomy revealed necrosis of the descending colon. This was
resected. Microscopic examination showed amorphous
eosinophilic material containing slit-like spaces occluding
arteries in the submucosa. Which one
of the following is the most likely explanation?
A. Thromboemboli secondary to myocardial infarction
B. Type B dissection of the aorta
C. Hypersensitivity vasculitis
D. Atheroembolism
E. Polyarteritis nodosa
The correct answer is D
Discussion
Atheroembolic disease increasingly is being recognized as an
important cause of morbidity and mortality in the developed world.
The true incidence of atheroembolism is not known, but it is likely
that many if not all adults older than 70 years will have evidence of
atheroembolism if it is sufficiently sought.
Even when the disease presents with the classic signs of digital
ischemia, livedo reticularis, and eosinophilia, other causes often are
sought. More often, patients present with a myriad of symptoms
because of cholesterol crystal deposition in small- and medium-
sized vessels. The clinical consequences range from pancreatitis to
penile gangrene. It is therefore not surprising that atheroembolism
has been described as a great mimicker. To make matters worse,
even when a diagnosis has been accurately established no
treatment has been shown to have a significant impact on the
attritional morbidity and mortality associated with atheroembolism
The clinical features of atheroembolism range from subclinical
disease to multisystem involvement leading to multiorgan failure
with an associated high mortality . The disease typically affects
white men 60 years of age and older. Established precipitants of
catastrophic atheroembolism are
o Preceding history of angiography or any instrumentation to the
aorta.
o Anticoagulant therapy.
o Cardiovascular surgery.
o Diabetes, hypertension, and previous vascular disease.
o Thrombolytic therapy (controversial (
A 57 year old man presents with a 35 minute history of severe chest
pain radiating down his left arm, associated with shortness of
breath. He denies denies a prior history of chest pain. His serum
troponin levels and EKG are compatible with an acute anterior
wall myocardial infarction. The histological finding you would
expect in the left anterior descending artery (LAD(
A. Circumferential medial fibrosis
B. Circumferential intimal fibrosis
C. Eccentric intact atherosclerotic plaques
D. Disrupted atherosclerotic plaque with non occlusive mural
thrombosis
E. Disrupted atherosclerotic plaque with occlusive thrombosis
Answer
E. Disrupted atherosclerotic plaque
with occlusive thrombosis
The rapid closure of coronary arteries
caused by occlusive thrombi is the major
cause of acute myocardial infarction.
Disruption of coronary atherosclerotic
plaques is recognized as one trigger of
coronary thrombosis. Formation of
thrombosis or hematoma may cause angina
pectoris or an acute coronary syndrome
due to occlusive thrombosis
 A 57 year old man presents with a 35 minute history of severe
chest pain radiating down his left arm, associated with
shortness of breath. He denies a prior history of chest pain. His
serum troponin levels and EKG are compatible with an acute
anterior wall myocardial infarction. The patient actually did
well initially, but collapsed outside his hospital room suddenly
6 days later. A new loud murmur indicating mitral insufficiency
was heard, but he died within 10 minutes. Of the following,
which is the most likely cause of death based on this history
A. Pulmonary embolism originating in left ventricular mural
thrombus
B. Ruptured ventricular papillary muscle
C. ruptured left ventricular papillary muscle
D. Acute bacterial endocarditis of mitral valve
E. Mitral valve prolapse
Answer
 C. ruptured left ventricular
papillary muscle
Ruptures can lead to papillary muscle
damage with acute mitral
insufficiency, which can lead to acute
hemodynamic changes on the left
side.
 The most common cause of death of in-
hospital patients with myocardial
infarctions
A. Cardiogenic shock (severe pump failure(
B. Arrhyrhmia
C. Cardiac free wall rupture
D. Rupture ventricular aneurysm
E. Constrictive pericarditis
Answer
B. Arrhyrhmia
 Ventricular fibrillation is the most common form of
arrhythmic death in acute myocardial infarction. The vast
majority of deaths due to ventricular fibrillation occur within
the first 24 h of the onset of symptoms, and of these
deaths, over half occur in the first hour.
 Most out-of-hospital deaths from myocardial infarction are
due to ventricular fibrillation. It may occur without warning
symptoms or arrhythmias.
 Over the last 30 years, with careful monitoring and prompt
attention to arrhythmias, the in-hospital mortality for acute
myocardial infarction has been reduced from about 30 to
between 10 and 15 percent.
 A 74 year old man presents with severe chest pain to the
emergency room, and is found to have a widened mediastinum
on chest radiograph and a murmur of aortic insufficiency on
examination. He dies on the operating table from blood loss
from a ruptured but very dilated ascending aorta. At autopsy,
severe aortic atherosclerosis, especially in the ascending
aorta, is seen together with a longitudinal intimal ridging
resembling tree bark affecting the ascending aorta. the aortic
root is also dilated. Histological sections of the aortic arch
confirm the atherosclerotic plaques, and also show
obliterative endarteritis of the vasa vasorum.
Your diagnosis:
A. Kawasaki arteritis; atherosclerosis
B. Atherosclerosis with secondary bacterial aortitis
C. Aortic dissection., Type A
D. Syphilitic aortitis with accelerated atherosclerosis
E. Takayasu arteritis
 D. Syphilitic aortitis with accelerated
atherosclerosis
Syphilitic aortitis causes obliterative
endarteritis of the vasa vasorum. This leads
to ischemia and smooth muscle atrophy of
the aortic media& can cause dilation of the
aortic ring leading to aortic insufficiency.
 A 93 year old man has a syncopal episode after being well most
of his life. Workup reveals aortic stenosis and you plan to
operate. You expect to find:
A. A calcified triscupid aortic valve without fused commisures; a
normal mitral valve for age

B. A calcified tricuspid aortic valve without fused commisures; a
normal mitral valve with fused, thickened chordae

C. A calcified tricuspid aortic valve with fused commisures; a
normal mitral valve for age

D. A calcified bicuspid aortic valve; a normal mitral valve for age

E. A calcified bicuspid aortic valve: a calcified pulmonic valve
Answer
A. A calcified triscupid aortic valve
without fused commisures; a
normal mitral valve for age
 Pulmonary hypertension is most severe in
which untreated valvular disease?
A. Pulmonary stenosis
B. Aortic stenosis
C. Aortic insufficiency
D. Mitral stenosis
E. Mitral insufficiency
Answer
 D. Mitral stenosis
Although not usually severe, the commonest forms of pulmonary
hypertension are those that occur secondary to chronic lung
disease and left ventricular failure.
 Thus, pulmonary hypertension and right ventricular hypertrophy
(cor pulmonale) may complicate the course of severe chronic
obstructive pulmonary disease (COPD).
 In addition, any cause of left ventricular failure may lead to
pulmonary venous congestion and hence pulmonary hypertension

Similarly, unidentified mitral stenosis may present with symptoms
of severe right heart failure. Other causes include congenital heart
disease, narrowing of small blood vessels in the lung, pulmonary
(emboli and primary pulmonary hypertension (PPH
 Constrictive pericarditis is most likely in
A. Uremia
B. Congestive heart failure
C. Rheumatic fever
D. Myocardial infarction
E. Tuberculosis
A body of a 57 year old woman underwent autopsy after she was
run down by the Graham Ambulance Service outside the clinic.
Multiple 2-4 mm vegetations were found on an otherwise normal
mitral valve. the vegetations were made of fibrin on histological
section, and the underlying valve was essentially normal. The
history that would be most likely when you get her clinic chart
A. Long history of intravenous drug abuse with recent history of
fevers to 104 degree F
B. Pancreatic carcinoma being treated on an experimental protocol
for chemotherapy
C. History of rheuthmatic fever at age nine with long term mitral
stenosis and recent onset of fevers to 103 degree F
D. Long history of systemic lupus erythematosis (SLE(
 D. Long history of systemic lupus erythematosis (SLE)
 Systemic lupus erythematosus (SLE) is an autoimmune
disease characterized by production of autoantibodies to
parts of the cell nucleus.
 Cardiac complications include pericarditis, endocarditis
(heart murmurs and valvular damage secondary to LIBMAN-
SACKS VEGETATIONS ON THE MITRAL VALVE), and coronary
artery disease (CAD). 50% of SLE patients have endothelial
damage to their heart valves
 Endocarditis in the form of nonbacterial vegetations (Libman
Sacks nodules) vary from mild valvular thickening (common
but with little or no functional problems) to large vegetations
affecting the function of the valve
 Severe back pain of acute onset in a 26 year old
woman in her 35th week of gestation for her third
child
A. Takayasu arteritis
B. Giant cell arteritis
C. Aortic dissection
D. Polyarteritis nodosa
E. Wegener's granulomatosis
 D. Polyarteritis nodosa
Polyarteritis nodosa (PAN) is a necrotizing systemic
vasculitis involving the wall of small and medium
sized arteries.
 The histologic aspect is defined by the presence of
fibrinoid necrosis and an infiltrate rich in neutrophil
polynuclears in the artery wall and rare granulomas.
 Clinical manifestations are misleading, and more often
the symptoms of the disease are retrospectively
related to the PAN.
 Which one of the following is not true about the development
of acute rheumatic fever?
 A. IT DEVELOPS DURING THE ACUTE PHASE OF A GROUP
A BETA-HEMOLYTIC STREPTOCOCCAL INFECTION OF
THE THROAT.
 B. IT IS NOT ASSOCIATEDWITHSTREPTOCOCCAL
INFECTION OF SITES OTHERTHAN THE PHARYNX.
 C. IT USUALLY OCCURS DURING THE COURSE OF
EPIDEMICS OF STREPTOCOCCAL THROAT INFECTIONS
IN CROWDEDSETTINGS.
 D. IT IS FARMORE COMMON IN UNDERDEVELOPED
COUNTRIES THAN IN THE UNITEDSTATES.
 E. ITS LESIONS INVOLVE THE HEART, JOINTS, SKIN AND
NERVOUS SYSTEM
 The Correct Answer is A
Acute rheumatic fever is triggered by infection with specific strains of
group A streptococci which possess antigens that cross-react with
human connective tissue, particularly heart valve glycoprotein...
The condition usually affects children or young adults, and
there is a familial variation in susceptibility...its prevalance in
Western Europe and North America has progressively declined
to very low levels, but it remains common in parts of asia, africa
and south america, where it is still the most common cause of
acquired heart disease in childhood adolescence...
Rheumatic fever is a systemic illness typically presenting with
fever, anorexia, lethargy and joint pains...
Arthritis occurs in approximately 75% of patients and other
features include skin rashes, carditis and neurological
features...
Anti-streptococcal antibody, anti-streptolysin antibody, positive
culture for group A streptococcus, recent scarlet fever...
Erythema marginatum (the expanding erythematous rash) and
sydenham chorea (rapid, purposeless movements) also
accompany this disorder...
Carditis is the most important manifestation of rheumatic
fever...carditis presents as breathlessness, palpitations or chest
pain...
 Otherfeatures consist of tachycardia, cardiac enlargement and
new orchanged cardiac murmurs...
A soft systolic murmuris common but non-specific...a soft mid-
diastolic murmuris often due to valvulitis, with nodules forming
on the mitral valve leaflets...
There is a pericardial friction rub which is oftenintermittent...
 cardiac failure may result eitherfromimpaired function of
ventricularmuscle orfrommitral oraortic incompetence and
tends to occurin a 'fulminant' formof rheumatic feverthat is
more common in developing countries...
Electrocardiographic changes include ST orT wave changes;
conduction defects sometimes occurand may cause syncope...
Mitral valve becomes thickened with fused chorda tendinea...
Aortic regurgitation occurs...
 Complications of rheumatic feverinclude:
 Cardiac arrhythmias
 Pericarditis
 Rheumatic pneumonitis
 Pulmonaryembolism
 Pulmonaryinfarction
 Valvedeformity, andinextremecases, congestiveheart
failure.
 The following are causes of prolonged QT
interval except:
A. Romano Ward syndrome
B. Erythromycin
C. Cisapride
D. Hypocalcaemia
E. Hyponatraemia
 The most common cardiac malformation
is:
A. Atrial septal defect
B. Endocardial cushion defect
C. Ventricularseptal defect
D. Aortic stenosis
 The correct answer is C
 VSD most common CHD(25%), membranous type most
common, large LA/RV, LV remains normal
 ASD
– ostiumsecundumtype most common, large RA/RV/PA ASD;
 PDA premature infants, maternal rubella, large LA/LV/aorta;
ESDostiumprimumASD, Down’s, gooseneckdeformity on
angio fromLV outlet obstruction
 PAPVR– right SVC most common, IVC (scimitarsyndro)
– most common cyanotic Tetralogy of Fallot
stenosis/RVH/VSD/overriding aorta, boot-shaped heart, right
aortic arch in 25%, small orconcave PA;
 Which one of the following statements is not true?
 A.THE VEGETATIONS OF INFECTIVE ENDOCARDITIS OFTEN
EMBOLIZE.
 B.THE VEGETATIONS OF NONBACTERIAL THROMBOTIC
ENDOCARDITIS OFTEN EMBOLIZE.
 C.THE VEGETATIONS OF ACUTE RHEUMATIC FEVER OFTEN
EMBOLIZE.
 D.THE VEGETATIONS OF SYSTEMIC LUPUS ERYTHEMATOSUS
(LIBMAN-SACKS DISEASE) LACK EMBOLIC PROPENSITY.
 E.FUNGAL ENDOCARDITIS VEGETATIONS ARE USUALLY VERY
LARGE.
 The Correct Answer is C
Small thrombi that develop on cardiac valves are termed vegetations.
Vegetations may be seen in bacterial infection of the valves (bacterial
endocarditis) also seen with IV drug users, systemic lupus
erythematosus, and in chronic wasting states.
 The four major forms of vegetative endocarditis:
A. RHD (Rheumatic Heart Disease) – Notice what looks like a string of
beads which are small verrucae or small vegetations, sterile deposits
of fibrin tend toward a predilection for the closing margin of the valve
(not the free edge). This is where the trauma is occurring as the valve
closes.
B. IE (Infective Endocarditis) – The damage may start on the closing
margin of the valve, particularly if the valve is already damaged.
Unlike RHD, they tend to be larger and bulkier. Particularly with fungal
infective endocarditis as compared to bacterial, the vegetations
become very large. The larger they are, the greater the risk of
breaking off and embolizing.
C. NBTE (non-bacterial thrombotic endocarditis) – tend to be localized
and also tend to be at the closing margin. They can grow large enough
to break off and embolize depending on the circumstances.
D. SLE (Libman-Sacks endocarditis) – Lesions also tend to occur at the
closing margin. One of the distinctive features here is that the
vegetations can be on either or both sides of the valve leaflets (notice
how the artist shows the leaflet partially lifted to demonstrate the
underside of the leaflet.
 Troponin T is a sensitive and specific marker
for:
 a. typical angina
b. crescendo angina
c. myocardial infarction
d. arrhythmia
e. Prinzmetal's angina
 The correct answer is C
Troponin T is a specific marker of myocardial
ischaemia and is sensitive providing the sample was
taken 16h or more after the suspected cardiac event;
earlier sampling may produce false negative results.
Troponin T may stay elevated for 7-10 days post MI.
In Acute Coronary Syndrome, Trop T >0.1 μg/L
consistent with myocardial infarction; Trop T 0.03 –
0.1 μg/L consistent with minor myocardial damage
 Complications of myocardial infarction
include all of the following except:
A. rupture of left ventricle
B. rupture of papillary muscle
C. mural thrombi
D. vasospasm
E. pericarditis
 The correct answer is D
Complications of Myocardial Infarction
 RECURRENT CHEST PAIN
 IF POST-MI ANGINA OCCURS (20-30%), PATIENTS
OFTEN GET PTCA OR CABG
 RECURRENT MI (5-20%) WITHIN FIRST SIX WEEKS
CARDIAC ARRHYTHMIAS AND CONDUCTION DEFECTS
(BRADY- AND TACHYARRHYTHMIAS)
 EXTREMELY COMMON W/ MI, MAJOR SOURCE OF
MORTALITY BEFORE PATIENT REACHES HOSPITAL
 Mechanisms
 ANATOMIC INTERRUPTION OF PERFUSION TO STRUCTURES OF
CONDUCTION PATHWAY
 ACCUMULATION OF LOCAL METABOLIC FACTORS AND
ABNORMAL TRANSCELLULAR ION CONCENTRATIONS DUE TO
MEMBRANE LEAKS
 AUTONOMIC STIMULATION
 ADMINISTRATION OF POTENTIALLY ARRYTHMOGENIC DRUGS
Types:
 VENTRICULAR FIBRILLATION
 DURING FIRST 48 HOURS PROBABLY RELATED TO ELECTRICAL INSTABILITY
 AFTER FIRST 48 HOURS PROBABLY RELATED TO LV DYSFUNCTION
 SUPRAVENTRICULAR ARRYTHMIAS
 SINUS BRADYCARDIA (EXCESSIVE VAGAL STIMULATION OR SA NODAL ISCHEMIA
 SINUS TACHYCARDIA (PAIN, ANXIETY, CHF, DRUGS, VOLUME DEPLETION)
 ATRIAL PREMATURE BEATS AND ATRIAL FIBRILLATION (ATRIAL ISCHEMIA OR
DISTENSION SECONDARY TO LV FAILURE)
 CONDUCTION BLOCKS
 MAY RESULT FROM ISCHEMIA OR INCREASED VAGAL TONE
 RIGHT VENTRICULAR INFARCTION
 1/3 OF THOSE WITH MI OF LV INFERIOR WALL WILL ALSO GET RV INFACT
B/C BOTH ARE SUPPLIED BY RIGHT CORONARY ARTERY (USUALLY(
 WILL GET SIGNS OF RIGHT SIDED HEART FAILURE INCLUDING JVD,
PROFOUND HYPOTENSION (LV BECOMES UNDERFILLED)
 MECHANICAL COMPLICATIONS, INCLUDING:
 PAPILLARY MUSCLE RUPTURE
 CAN BE RAPIDLY FATAL BECAUSE OF SEVERE MITRAL REGURGITATION
 VENTRICULAR FREE WALL RUPTURE
 MAY OCCUR WITHIN FIRST 2 WEEKS OF MI
 MORE COMMON IN WOMEN AND THOSE WITH A HX OF HYPERTENSION
 RESULTS IN CARDIAC TAMPONADE, PSEUDOANEURYSM (IF RUPTURE IS
INCOMPLETE AND PLUGGED WITH A THROMBUS)
 VENTRICULAR SEPTAL RUPTURE
1. VENTRICULAR SEPTAL RUPTURE
DEVELOP LEFT TO RIGHT SHUNT
PRECIPITATES CHF BY VOLUME OVERLOAD OF PULMONARY
CAPILLARIES
1. VENTRICULAR ANEURYSM
 USUALLY OCCURS WEEKS TO MONTHS AFTER MI AS WALL
PROGRESSIVELY WEAKENS, BUT NOT PEFORATED BY
PHAGOCYTIC CLEARANCE OF NECROTIC TISSUE
 GET LOCALIZED OUTWARD BUGLE WHEN VENTRICLE
CONTRACTS
 RUPTURE AND TAMPONADE DO NOT DEVELOP
 COMPLICATIONS INCLUDE THROMBUS FORMATION,
VENTRICULAR ARRHYTHMIAS, HEART FAILURE
 GET PERSISTENT ST SEGMENT ELEVATION (WEEKS AFTER Q
WAVE MI
1. “PUMP” FAILURE
 IMPAIRED VENTRICULAR CONTRACTILITY AND INCREASED
MYOCARDIAL STIFFNESS BOTH LEAD TO SYMPTOMS OF HEART
FAILURE
 SIGNS AND SYMPTOMS INCLUDE DYSPNEA, PULMONARY RALES, S3
 TREATMENT INCLUDES DIURESIS AND VASODILATOR THERAPY
 A patient has persist >2mm ST elevation in V2-6 two
hours following a myocardial inraction, with hypertesnion
of 205/115, he has already been given morphine and
aspirin , what is the next management of choice?
a) IV Nitroglycerine
b) Double-bolus r-PA
c) iv GTN
d) iv streptokinase
e) subcutaneous heparin
 The correct answer is A
1) patients with ST-segment elevation, i.e. new ST-segment
elevation at the J point with the cut-off points 0.2mV in
V1through V3and 0.1mV in other leads, or
1) patients without ST-segment elevation, i.e. ST-segment
depression or T wave abnormalities.
 Clinically established myocardial infarction may be defined by
any Q wave in leads V1through V3, or Q wave 0.03s in leads I,
II, aVL, aVF, V4, V5or V6.
 Neither the GISSI-2/International Trials nor the Third
International Study of Infarct Survival(ISIS 3)found a
difference in mortality between the use of streptokinase and
tissue plasminogen activator or anistreplase. Furthermore, the
addition of subcutaneous heparin did not reduce mortality
compared with the use of no heparin.
 However, the GUSTO Trial (Global Utilisation of Streptokinase
and Tissue Plasminogen Activator for occluded coronary
arteries)28 employed an accelerated t-PA (tissue type
plasminogen activator) regimen given over 90min29 rather
than the previously conventional period of 3h. Accelerated t-PA
with concomitant APTT (activated partial thromboplastin time)
adjusted intravenous heparin was reported to result in 10 fewer
deaths per 1000 patients treated. The risk of stroke is higher
with t-PA or anistreplase than with streptokinase.
 In the GUSTO trial, there were three additional strokes per
1000 patients treated with accelerated t-PA and heparin in
comparison with streptokinase and subcutaneous heparin, but
only one of these survived with a residual deficit.
 In assessing the net clinical benefit, this must be taken into
account with the reduced death rate in the t-PA group. Several
variants of t-PA have been studied. Double-bolus r-PA
(reteplase) does not offer any advantage over accelerated t-PA
except for its ease of administration. Single-bolus weight-
adjusted TNK-tPA (tenecteplase) is equivalent to accelerated t-
PA for 30-day mortality and associated with a significantly
lower rate of non-cerebral bleeds and less need for blood
transfusion.
 Bolus fibrinolytic therapy may facilitate more rapid
treatment in and out of the hospital and reduce the risk
of medication errors. The choice of fibrinolytic agent will
depend on an individual assessment of risk and benefit,
and also on factors such as availability and cost. For late
treated patients more fibrin-specific agents may be
more effective
 Platelet aggregation is only partly inhibited by aspirin and progress has been made
with the development of platelet glycoprotein IIb/IIIa inhibitors, which block the
final pathway of platelet aggregation. Angiographic trials demonstrated that the
combination of GP IIb/IIIa inhibitors with half-dose fibrinolytic and reduced doses
of heparin, induces similar or slightly higher TIMI grade 3 flow rates when
compared with full-dose fibrinolytic alone and is associated with more complete
resolutionof ST-segment elevations, suggesting an improvement in tissue
reperfusion. The clinical benefit and safety of these combinations has been

 tested in two large trials.No reductions in 30-day mortality or intracranial
haemorrhage rates but lower rates of in-hospital reinfarction were observed,
however, at the cost of an increase in (mostly spontaneous) non-cerebral bleeding
complications especially in elderly patients.
 Therefore, the routine use of a reduced dose fibrinolytic with abciximab or other
platelet glyco-protein IIb/IIIa inhibitors cannotbe recommended. Whether this
combination therapy may be beneficial in specific subgroups of patients (for
example those at high risk or those likely to undergo early PCI) needs to be further
evaluated.
 Heparin has been extensively used during and after fibrinolysis, especially with
tissue plasminogen activator. Heparin does not improve immediate clot lysis but
coronary patency evaluated in the hours or days following thrombolytic therapywith
tissue plasminogen activator appears to be better with intravenous heparin. No
difference in patency was apparent in patients treated with either subcutaneous or
intravenous heparin and streptokinase.Prolonged intravenous heparin
administration has not been shown to prevent reocclusion after angiographically
proven successful coronary fibrinolysis. Heparin infusion after tissue plasminogen
activator therapy may be discontinued after 24–48h. Close monitoring of
intravenous heparin therapy is mandatory; aPTT values over 70s are associated
with higher likelihood of mortality, bleeding and reinfarction. Although no
randomized trials have beenperformed, there is evidence from recent trials
suggesting that more frequent monitoring of aPTT and a full weight adjustment of
heparinmay decrease the risk of non-cerebral bleeding complications.

 Familial hypertrophic cardiomyopathy is most
likely to be secondary to a mutation in:
a) myosin regulatory proteins
b) myosin binding protein-C
c) Myosin light chains
d) Troponin I
e) Troponin T
 The correct answer is B
Molecular genetic research has demonstrated that familial
hypertrophic cardiomyopathy is caused by a mutation in one of
nine genes that encode sarcomere proteins.
 Sarcomeres are the contractile units within the cardiac cells;
these are composed of many proteins that are organized into
thin and thick filaments. These filaments slide with respect to
each other during cardiac contraction.
Mutations in thick filament proteins called cardiac B myosin
heavy chain or cardiac myosin binding protein-C appear to
account for approximately 82% of hypertrophic
cardiomyopathy. Mutations in thin filament proteins cardiac
troponin T and a tropomyosin account for about 13% of
hypertrophic cardiomyopathy. Mutations in two other
sarcomere proteins, the myosin regulatory and essential light
chains are quite rare and contribute less than 5% to
hypertrophic cardiomyopathy. Although these percentages are
only estimates, they indicate that more disease-causing genes
will be identified.
A tenth gene has been identified that is a non-sarcomere
protein (a subunit of protein kinase A) is associated with
individuals who have both hypertrophic cardiomyopathy and
Wolf-Parkinson-White syndrome.
 A 42-year-old man with acute renal failure is
confused. His serum potassium is 8.1 mEq/L . The
most likely abnormal ECG finding is
 a) T wave inversion
b) PR interval of 300ms
c) QT interval of 0.4s
d) U wave
e) tall tented T waves test
 The correct answer is E
 the earliest ECG evidence of hyperkalemia usually
appears in the T waves The variety of changes
include:
 Increased amplitude and peaking of the T wave
 PR interval prolongation
 QRS interval prolongation Flattening of the P wave.
 A plasma potassium of >6.5mmol/l should be treated
urgently unless it is an artefact. Hyperkalemia may
have a variety of causes: Renal failure; Excess
potassium replacement therapy;Acidosis from any
one of various causes (diabetic ketoacidosis, lactic
acidosis, etc.); Presence of insufficient
corticosteroids (Addison's disease). As in
hypokalemia, there may be a poor correlation
between serum potassium levels and the typical ECG
changes.
 Which of the following is most commonly associated with
an increased risk for the development of torsades de
pointes?
a) decreased PR interval
b) decreased QRS duration
c) increased PR interval
d) increased QT interval
e) U waves
 The correct answer is D

 There is an increased risk for the development of torsades de
pointes, a potentially fatal arrhythmia, when the QT interval is
prolonged. Roden reviewed the drugs most commonly
implicated in QT prolongation and the clinical factors that
increase the risk of torsades de pointes.
 Medication-induced QT prolongation was first recognized with
the use of quinidine in the 1920s. Roden lists other drugs that
also are implicated in prolongation of the QT interval and may
cause torsades de pointes . Use of these medications, especially
when a congenital QT prolongation syndrome or other clinical
risk factor is present, increases the chance that torsades de
pointes may develop. Because the risk of torsades de pointes is
sufficiently high at typical clinical dosages of sotalol, dofetilide,
and ibutilide, the review author recommends inhospital cardiac
monitoring when therapy with these agents is initiated
 The risk of torsades de pointes is not related linearly to the
degree of QT prolongation, although any drug that prolongs the
QT interval beyond 500 msec is thought to confer an elevated
risk. Heart rate exerts an important effect on the risk for
associated torsades de pointes, with a greater propensity for
developing the arrhythmia when bradycardia is present.

 A 21 year old healthy female comes to the clinic with
complaints of intermittent palpitations. These episodes
can last from seconds to minutes and are associated
with lightheadedness, diaphoresis, and occassionally
chest pain. Her ECG is below:
What are the ECG finding?
 Normal sinus rhythm
2) Short PR interval and delta waves consistent with Wolff-
Parkinson-White (WPW) syndrome

Explanation: WPW occurs when a person is born with an
"accessory pathway" that can bypass the AV node and conduct
action potentials straight from the atrium to the ventricle.
 Normally, the AV node acts to delay the action potentials created in
the sinus node before they reach the ventricles. This normal delay
is represented by the PR interval (the time it takes for the sinus
nodal action potential, which creates the P wave, to reach the
ventricles and cause ventricular depolarization, which creates the
QRS complex). An accessory pathway can conduct action potentials
very fast without any delay (unlike the AV node). Thus in WPW,
after the sinus node fires creating an action potential, conduction
immediately goes to the ventricles through the accessory pathway
without delay creating a short PR interval (PR interval < 0.12
seconds).
 Since the PR interval is so short, the P wave is very close to the
QRS complex and the two usually blend together making the QRS
complex appear to begin with an upslope. This upslope is called a
"delta wave" which is pathogneumonic for the presence of an
accessory pathway indicating WPW syndome
What medications should be avoided in this patient?
AV nodal blocking medications
 WPW occurs when a person is born with an "accessory
pathway" that can bypass the AV node and conduct
action potentials straight from the atrium to the
ventricle.
 Since the accessory pathway has a very short refractory
period, it can conduct impulses from the atria to the
ventricles at very fast rates (> 400 imulses per minute).
If the AV nodal conduction is blocked using the AV nodal
blocking medications, more conduction can occur
through the accessory pathway.
 In certain arrythmias like atrial fibrillation this can
cause the ventricular rate to become dangerously fast.
 A good pneumonic to remember the AV nodal blocking
medications is "ABCD" which stands for Adenosine, Beta-
blockers, Calcium channel blockers, and Digoxin.
In your clinic she has one of these episodes and an ECG is
performed
What are the ECG #2 findings?
 Narrow complex tachycardia consistent with atrial
fibrillation and WPW syndrome (less likely AVNRT,
although still possible.
What medication should be given if the rhythm caused
hemodynamic instability or cardiac arrest?
 Procainamide
Once again the AV nodal blocking medications should not
be used since blocking the AV node allows more impuses to
travel through the accessory pathway thus making the
ventricular rate dangerously fast, often precipitating
ventricular fibrillation which is fatal. Procainamide actually
slows conduction through the accessory pathway while
increasing the conduction through the AV node and is the
treatment of choice in patients with WPW and suspected
atrial fibrillation.
Patients with WPW are also at an increased risk of
developing AV nodal rentry tachycardias (AVNRT) which are
difficult to distinguish from atrial fibrillation on ECG due to
the fast ventricular rate present in both rhythms making it
hard to determine if it is irregular. Adenosine is actually
relatively safe to give if the rhythm is AVNRT since no
conduction is occuring in the accessory pathway as occurs
with the combination of WPW and atrial fibrillation
(remember AVNRT occurs with dual pathways within the AV
node
 A 55 year old male with a history of diabetes mellitus
type II and hypertension presents to your clinic and has
the below ECG
What are the ECG findings?
 The ECG findings include:
Normal sinus rhythm
Poor R wave progression (PRWP)
What is the differential diagnosis of this ECG finding?
 The differential diagnosis for poor R wave
progression (PRWP) includes :
 Old anteroseptal infarct (Q waves in V1 to v3)
Left ventricular hypertrophy
Left anterior fascicular block
Left bundle branch block
Late transition or clockwise rotation of the heart
(normal varient)
Wolff-Parkinson-White
 You are the house officer on call for the
telemetry floor and a nurse calls you with an
abnormal rhythm seen on telemetry. The
patient was admitted 2 days previously for a
congestive heart failure exacerbation. He
denies any symptoms including chest pain,
palpitations, lightheadedness, or shortness of
breath. Vital signs are normal. You ask for a
12-lead ECG which is below:
What are the ECG finding?
 The ECG findings include:
Normal sinus rhythm
 Prominent U waves
A "U wave" occurs just after the T wave and
significantly before the following P wave. U waves can
at times be mistaken for atrial flutter waves or
additional P waves indicating AV block. Also, if the U
wave overlaps with the T wave it may mimic a
prolonged QT interval
.
What is the electrophysiologic mechanism responsible for
the findings in the above ECG?
 Prolonged depolarization of a portion of the
ventricles
It is though that a special subpopulation of myocardial
cells exists in the mid-myocardium that has longer action
potentials and longer repolarization times.
 With the presence of specific stressors (hypokalemia,
bradycardia, digoxin use, or LVH), amplitude of the T
wave in the special subpopulation of myocardium with a
longer repolarization times increases.
 Since this repolarization time is significantly longer than
normal myocardium (in which repolarization creates the
T wave), a distinct wave after the T wave is produced
which is called the U wave.
You go back to sleep and are soon called again for another abnormal
rhythm seen on the same patient which is intermittently occurring.
The patient is still asymptomatic and vital signs are still normal.
Another 12-lead ECG is obtained which is below
Dx
 The ECG findings include:
Normal sinus rhythm
Non-sustained monomorphic ventricular tachycardia
with a rate of slightly less than 300.
 Non-sustained ventricular tachycardia (NSVT) can occur for
multiple reasons, however it is very commonly seen in
patients with systolic congestive heart failure.
 Short runs of NSVT are usually asymptomatic, however
if the NSVT becomes sustained it can be life-threatening.
 Sustained ventricular tachycardia is usually defined as
VT lasing for more than 30 seconds. VT can be
monomorphic (most common) or polymorphic (a.k.a.
Torsades de Pointes(
TX
 Replace electrolytes and consider defibrillator
placement if criteria met
 NSVT alone is not life-threatening, however it predicts
an increased risk of sudden cardiac death.
 Correcting electrolyte abnormalities or
revascularizing the treat ischemia significantly
reduces the VT.
 Evidence supports the implantation of automated
implanted cardiac defibrillators (AICDs) in all patients
with systolic congestive heart failure with an ejection
fraction of < 35%.
 An AICD can quickly electrically cardiovert a patient
should sustained VT occur.
45year-old man with a strong family history of ischaemic
heart disease presents with atypical chest pains. Stress
echocardiography is organised. What pharmacologic
agent is likely to be used produce cardiovascular stress
during Stress echocardiography in this patient?
a) dipyridamole
b) Dobutamine
c) adenosine
d) arbutamine
e) Atropine sulfate
 The correct answer is A
Explanation
 Stress echocardiography has been developed in recent years as an
effective noninvasive test for the detection and assessment of coronary
artery disease. This method combines exercise with 2-dimensional
echocardiography, which can assess regional and global left ventricular
function during stress.
 Dobutamine infusion, a pharmacologic means of producing
cardiovascular stress, appears to be an excellent alternative to exercise in
echocardiographic studies. Currently, it is reserved for patients who
cannot exercise at a meaningful level because of advanced age, physical
deconditioning, or other factors.
 Dobutamine infusion is the method used most often for pharmacologic
stress echocardiography. Graded dobutamine infusion--10 to 40
micrograms/kg per minute in 3-minute stages--increases myocardial
oxygen demand in a fashion similar to that of staged exercise. During the
dobutamine infusion, it is apparent that heart rate, contractility, and
blood pressure are increased. Dobutamine has the advantage of rapid
onset of action, and its effects can be reversed by giving an intravenous
beta-blocker.
 A synthetic catecholamine that has a relatively short half-life (about 2
minutes) , dobutamine has strong agonist activity at the beta1 receptor
and mild agonist activity at the beta2 and alpha1 receptors.
 Atropine sulfate can be used to increase heart rate, if necessary, and is usually
administered at the peak dobutamine dose. It is usually given as a 0.5-mg bolus and
in 0.25-mg increments every 60 seconds (maximum dose, 1-1.5 mg) until the
desired heart rate is achieved.
Dobutamine infusion is stopped after images are acquired at peak heart rate--or
sooner if the patient has tachyarrhythmias.
 A 65 year old male with end-stage COPD gets admitted
to the ICU with respiratory failure. He was found to
have the below ECG
DX
 The ECG findings include:
Multifocal atrial tachycardia (three distinct P wave
morphologies)
Incomplete right bundle branch block
Poor R wave progression
PVCs
Multifocal atrial tachycardia (aka MAT) is an irregularly irregular,
tachycaric rhythm in which many foci in the atium chaotically fire
acting as the pacemaker of the heart instead of the sinus node.
The atrial rate is not as fast as in atrial flutter or atrial fibrillation,
so normal AV synchrony can occur. When the rhythm has 3 distinct
P wave morphologies and the heart rate is not fast, the term
"wandering atrial pacemaker" or WAP is used.
TX
 The treatment of multifocal atrial tachycardia is
aimed at the underlying cause.
 In our case it would be to treat the COPD
exacerbation and respiratory failure.
 The only medication that has been used to
treat MAT is verapamil with only marginal
success.
 No anticoagulation is needed for MAT in
contrast to atrial flutter/fibrillation since the
atrium are contracting well, but simply
originating in different areas.
 Hemoptysis, hematuria and recent nosebleeds in a 43
year old man:
A. Takayasu arteritis
B. Giant cell arteritis
C. Aortic dissection
D. Polyarteritis nodosa
E. Wegener's granulomatosis
 E. Wegener's granulomatosis
 This condition is uncommon and occurs in young and
middle aged adults, with a slight male predominance.
The vessels mainly involved in this disease are the
small arteries and veins. Necrotizing granulomatous
lesions of the respiratory tract, focal segmental
glomerulonephritis and vasculitis of other organs
characterizes this condition.

Presenting features include non-specific findings
such as fever, weight loss
 myalgias, arthralgias, malaise and chronic rhinitis or
sinusitis. Supportive otitis, mastoiditis, hearing loss
and a saddle-nose defect may occur.
 Jones criteria for the diagnosis of acute rheumatic fever
include major and minor criteria. In the presence of
evidence of a preceding group A streptococcal infection,
which one of the following statements is correct?
 a. the diagnosis requires the presence of one major
criterion only.
 b. the diagnosis requires the presence of at least
three major criteria.
 c. the diagnosis requires the presence of two major
or one major and two minor criteria.
 d. three minor criteria alone are sufficient for the
diagnosis.
 e. two major criteria in the absence of minor criteria
are insufficient for the diagnosis
 The correct answer is C
For Diagnosis of Rheumatic fever, two major criteria or one
major and two minor criteria must be present.
 * Major Jones Criteria (manifestations) of Rheumatic fever
. Carditis
. Polyarthritis
. Chorea
. Erythema marginatum
. Subcutaneous nodules
 * Minor Jones criteria of Rheumatic fever:
Clinical:
- Previous rheumatic fever or rheumatic heart disease
- arthralgia
- Fever
Laboratory
-↑Erythrocyte sedimentation rate (ESR)
-↑C-reactive protein
 A patient develops an acute febrile illness with shivers,
nonproductive cough, and pleuritic chest pain. Five days
later, he presents to the emergency room after abruptly
having "coughed up" nearly a cup of
blood-stained sputum. Which of the following is most likely
to be seen on chest x-ray?
A. A cavity with a fluid level
B. Blunting of diaphragmatic costal angles
C. Complete opacification of one lobe with no additional
findings
D. Patchy consolidation centered on bronchi
E. Prominent bronchi that can be followed far out into the
lung fields
 The correct answer is A.
This is a classic presentation of a pulmonary abscess.
Chronic courses with less severe symptoms (with intermittent
improvement following short courses of antibiotics) are
also sometimes seen, particularly if the diagnosis was not
suspected.
Chest x-ray typically shows pneumonic opacification in which a
cavity, often with a fluid level, is visible.
Pulmonic abscesses can be caused by anaerobes (most
common, particularly if aspiration initiated the abscess),
gram-negative aerobic bacilli, and Staphylococci.
Therapy is based on the organisms isolated, and should be
continued for at least 4 to 6 weeks.
In cases that fail to resolve, the possibility of coexisting
carcinoma should be considered.
Choice B is the x-ray appearance of pleural effusion.
Choice C is the x-ray appearance of lobar pneumonia.
Choice D is the x-ray appearance of bronchopneumonia.
Choice E is the x-ray appearance of bronchiectasis.
 Which congenital heart disease is most likely to
have hypertrophic and dilated bronchial arteries
as a compensatory mechanism?
a. ventricular septal defect
b. aortic stenosis
c. atrial septal defect
d. persistent truncus arteriosus
e. tetralogy of Fallot
 The Correct Answer is E
 In T.O.F.
Roentgenogrophic examination characteristically reveals a
normal-sized, boot-shaped heart (“coeur en sabot”) with
prominence of the right ventricle and a concavity in the region
of the underdeveloped right ventricular outflow tract and main
pulmonary artery.
The pulmonary vascular markings typically are diminished, and
the aortic arch and knob may be on the right side; the
ascending aorta usually is large. A uniform, diffuse, fine
reticular pattern of vascular markings is noted in the presence
of prominent collateral vessels .
The degree of obstruction to pulmonary blood flow is the
principal determinant of the clinical presentation. The site of
obstruction is variable. Infundibular stenosis is the only major
obstruction in about 50% of patients and coexists with valvular
obstruction in another 20 to 25 per cent.
Supravalvular and peripheral pulmonary arterial narrowing may
be observed, and unilateral absence of a pulmonary artery
(usually the left) is found in a small number of patients.
Circulation to the abnormal lung is accomplished by bronchial
and other collateral arteries.
THANKS FOR YOUR PATIENTS
‫ونفعت‬ ‫أثمرت‬ ‫تكون‬ ‫أن‬ ‫عسى‬ ---- ‫دعواتكم‬

Mais conteúdo relacionado

Mais procurados

Hypertrophic cardiomyopathy
Hypertrophic cardiomyopathyHypertrophic cardiomyopathy
Hypertrophic cardiomyopathyDr.Deepika T
 
Dilated cardiomyopathy
Dilated cardiomyopathyDilated cardiomyopathy
Dilated cardiomyopathyAvinash Arke
 
Dilated cardiomyopathy
Dilated cardiomyopathyDilated cardiomyopathy
Dilated cardiomyopathyAvinash Arke
 
Pulmonary Oedema - Pathophysiology - Approach & Management
Pulmonary Oedema  - Pathophysiology - Approach & ManagementPulmonary Oedema  - Pathophysiology - Approach & Management
Pulmonary Oedema - Pathophysiology - Approach & ManagementArun Vasireddy
 
Mitral regurgitation
Mitral regurgitationMitral regurgitation
Mitral regurgitationDiaa Srahin
 
500 single best answers in medicine
500 single best answers in medicine500 single best answers in medicine
500 single best answers in medicinehamadadodo
 
Sudden cardiac death
Sudden cardiac deathSudden cardiac death
Sudden cardiac deathSunil Reddy D
 
Internal Medicine Sample Questions
Internal Medicine Sample QuestionsInternal Medicine Sample Questions
Internal Medicine Sample QuestionsDJ CrissCross
 
Refractory heart failure - Diagnosis, Management, Device Therapy
Refractory heart failure - Diagnosis, Management, Device TherapyRefractory heart failure - Diagnosis, Management, Device Therapy
Refractory heart failure - Diagnosis, Management, Device TherapyImran Ahmed
 
Acute Myocarditis:Diagnosis and Management
Acute Myocarditis:Diagnosis and ManagementAcute Myocarditis:Diagnosis and Management
Acute Myocarditis:Diagnosis and ManagementPawan Ola
 
Diagnosis and management of acute heart failure
Diagnosis and management of acute heart failureDiagnosis and management of acute heart failure
Diagnosis and management of acute heart failureAlaa Ateya
 
Pericardial diseases
Pericardial  diseasesPericardial  diseases
Pericardial diseasesikramdr01
 
Supraventricular tachycardias
Supraventricular tachycardiasSupraventricular tachycardias
Supraventricular tachycardiasPraveen Nagula
 

Mais procurados (20)

Hypertrophic cardiomyopathy
Hypertrophic cardiomyopathyHypertrophic cardiomyopathy
Hypertrophic cardiomyopathy
 
Dilated cardiomyopathy
Dilated cardiomyopathyDilated cardiomyopathy
Dilated cardiomyopathy
 
Dilated cardiomyopathy
Dilated cardiomyopathyDilated cardiomyopathy
Dilated cardiomyopathy
 
Pulmonary Oedema - Pathophysiology - Approach & Management
Pulmonary Oedema  - Pathophysiology - Approach & ManagementPulmonary Oedema  - Pathophysiology - Approach & Management
Pulmonary Oedema - Pathophysiology - Approach & Management
 
Heart failure
Heart failureHeart failure
Heart failure
 
Mitral stenosis
Mitral stenosisMitral stenosis
Mitral stenosis
 
ENDOMYOCARDIAL FIBROSIS
ENDOMYOCARDIAL FIBROSISENDOMYOCARDIAL FIBROSIS
ENDOMYOCARDIAL FIBROSIS
 
Mitral regurgitation
Mitral regurgitationMitral regurgitation
Mitral regurgitation
 
500 single best answers in medicine
500 single best answers in medicine500 single best answers in medicine
500 single best answers in medicine
 
Sudden cardiac death
Sudden cardiac deathSudden cardiac death
Sudden cardiac death
 
Takotsubo Cardiomyopathy
Takotsubo CardiomyopathyTakotsubo Cardiomyopathy
Takotsubo Cardiomyopathy
 
Internal Medicine Sample Questions
Internal Medicine Sample QuestionsInternal Medicine Sample Questions
Internal Medicine Sample Questions
 
HOCM Hypertrophic cardiomyopathy
HOCM Hypertrophic cardiomyopathyHOCM Hypertrophic cardiomyopathy
HOCM Hypertrophic cardiomyopathy
 
Refractory heart failure - Diagnosis, Management, Device Therapy
Refractory heart failure - Diagnosis, Management, Device TherapyRefractory heart failure - Diagnosis, Management, Device Therapy
Refractory heart failure - Diagnosis, Management, Device Therapy
 
Acute Myocarditis:Diagnosis and Management
Acute Myocarditis:Diagnosis and ManagementAcute Myocarditis:Diagnosis and Management
Acute Myocarditis:Diagnosis and Management
 
Diagnosis and management of acute heart failure
Diagnosis and management of acute heart failureDiagnosis and management of acute heart failure
Diagnosis and management of acute heart failure
 
Pericardial diseases
Pericardial  diseasesPericardial  diseases
Pericardial diseases
 
Supraventricular tachycardias
Supraventricular tachycardiasSupraventricular tachycardias
Supraventricular tachycardias
 
Dilated cardiomyopathy
Dilated cardiomyopathyDilated cardiomyopathy
Dilated cardiomyopathy
 
Cardiomyopathy
CardiomyopathyCardiomyopathy
Cardiomyopathy
 

Destaque

Cardiology board mc qs ppt.ppt7
Cardiology board mc qs ppt.ppt7Cardiology board mc qs ppt.ppt7
Cardiology board mc qs ppt.ppt7hospital
 
Clinical Pearls in Cardiology
Clinical Pearls in CardiologyClinical Pearls in Cardiology
Clinical Pearls in CardiologyMadhusree Singh
 
Cardiology board mc qs
Cardiology board mc qsCardiology board mc qs
Cardiology board mc qshospital
 
Mcq 1060 questions
Mcq 1060 questionsMcq 1060 questions
Mcq 1060 questionsadrioz
 
Dr magdi sasi mcq in medicine part one
Dr magdi  sasi    mcq  in medicine  part oneDr magdi  sasi    mcq  in medicine  part one
Dr magdi sasi mcq in medicine part onecardilogy
 
Cardiology board mc qs.ppt2
Cardiology board mc qs.ppt2Cardiology board mc qs.ppt2
Cardiology board mc qs.ppt2hospital
 
Cardiology board mc qs.ppt1
Cardiology board mc qs.ppt1Cardiology board mc qs.ppt1
Cardiology board mc qs.ppt1hospital
 
SCLERODERMA DR MAGDI AWAD SASI 2016 LMB
SCLERODERMA DR MAGDI AWAD SASI 2016 LMBSCLERODERMA DR MAGDI AWAD SASI 2016 LMB
SCLERODERMA DR MAGDI AWAD SASI 2016 LMBcardilogy
 
Chronic renal failure by dr m.s. magdi awad sasi(( part 3 -- renal failure))
Chronic renal failure  by dr m.s. magdi awad sasi(( part 3 -- renal failure))Chronic renal failure  by dr m.s. magdi awad sasi(( part 3 -- renal failure))
Chronic renal failure by dr m.s. magdi awad sasi(( part 3 -- renal failure))cardilogy
 
Cases in INTERNAL MEDICINE part one PART FIFTH DR MAGDI SASI
Cases in INTERNAL MEDICINE part one PART FIFTH DR MAGDI SASICases in INTERNAL MEDICINE part one PART FIFTH DR MAGDI SASI
Cases in INTERNAL MEDICINE part one PART FIFTH DR MAGDI SASIcardilogy
 
Cardiology board images
Cardiology board imagesCardiology board images
Cardiology board imageshospital
 
Dr magdi sasi cns examination 1
Dr magdi sasi  cns  examination 1Dr magdi sasi  cns  examination 1
Dr magdi sasi cns examination 1cardilogy
 
Cardiology lecture toIternal Medicine 21/10/2013
Cardiology lecture toIternal Medicine 21/10/2013Cardiology lecture toIternal Medicine 21/10/2013
Cardiology lecture toIternal Medicine 21/10/2013hospital
 
Cases in cardiology part one PART FOUR 2016--
Cases in cardiology part one PART FOUR 2016--Cases in cardiology part one PART FOUR 2016--
Cases in cardiology part one PART FOUR 2016--cardilogy
 
Cardiology presentation 2 im 762012
Cardiology presentation 2 im 762012Cardiology presentation 2 im 762012
Cardiology presentation 2 im 762012hospital
 
Cardio respiratory nuclear imaging ihab - copy
Cardio respiratory nuclear imaging ihab - copyCardio respiratory nuclear imaging ihab - copy
Cardio respiratory nuclear imaging ihab - copyhospital
 
ACUTE and CHRONIC AORTIC INSUFFICIENCY-DR MAGDI SASI 2016
ACUTE and CHRONIC AORTIC INSUFFICIENCY-DR MAGDI SASI 2016ACUTE and CHRONIC AORTIC INSUFFICIENCY-DR MAGDI SASI 2016
ACUTE and CHRONIC AORTIC INSUFFICIENCY-DR MAGDI SASI 2016cardilogy
 
DBMS Multiple Choice Questions
DBMS Multiple Choice QuestionsDBMS Multiple Choice Questions
DBMS Multiple Choice QuestionsShusil Baral
 
rheumatic_feve for dentist 201`6--DR MAGDI SASI
rheumatic_feve for dentist 201`6--DR MAGDI SASIrheumatic_feve for dentist 201`6--DR MAGDI SASI
rheumatic_feve for dentist 201`6--DR MAGDI SASIcardilogy
 
Ssc mcq-question-ans-2-motion
Ssc mcq-question-ans-2-motionSsc mcq-question-ans-2-motion
Ssc mcq-question-ans-2-motionSumon Hossain
 

Destaque (20)

Cardiology board mc qs ppt.ppt7
Cardiology board mc qs ppt.ppt7Cardiology board mc qs ppt.ppt7
Cardiology board mc qs ppt.ppt7
 
Clinical Pearls in Cardiology
Clinical Pearls in CardiologyClinical Pearls in Cardiology
Clinical Pearls in Cardiology
 
Cardiology board mc qs
Cardiology board mc qsCardiology board mc qs
Cardiology board mc qs
 
Mcq 1060 questions
Mcq 1060 questionsMcq 1060 questions
Mcq 1060 questions
 
Dr magdi sasi mcq in medicine part one
Dr magdi  sasi    mcq  in medicine  part oneDr magdi  sasi    mcq  in medicine  part one
Dr magdi sasi mcq in medicine part one
 
Cardiology board mc qs.ppt2
Cardiology board mc qs.ppt2Cardiology board mc qs.ppt2
Cardiology board mc qs.ppt2
 
Cardiology board mc qs.ppt1
Cardiology board mc qs.ppt1Cardiology board mc qs.ppt1
Cardiology board mc qs.ppt1
 
SCLERODERMA DR MAGDI AWAD SASI 2016 LMB
SCLERODERMA DR MAGDI AWAD SASI 2016 LMBSCLERODERMA DR MAGDI AWAD SASI 2016 LMB
SCLERODERMA DR MAGDI AWAD SASI 2016 LMB
 
Chronic renal failure by dr m.s. magdi awad sasi(( part 3 -- renal failure))
Chronic renal failure  by dr m.s. magdi awad sasi(( part 3 -- renal failure))Chronic renal failure  by dr m.s. magdi awad sasi(( part 3 -- renal failure))
Chronic renal failure by dr m.s. magdi awad sasi(( part 3 -- renal failure))
 
Cases in INTERNAL MEDICINE part one PART FIFTH DR MAGDI SASI
Cases in INTERNAL MEDICINE part one PART FIFTH DR MAGDI SASICases in INTERNAL MEDICINE part one PART FIFTH DR MAGDI SASI
Cases in INTERNAL MEDICINE part one PART FIFTH DR MAGDI SASI
 
Cardiology board images
Cardiology board imagesCardiology board images
Cardiology board images
 
Dr magdi sasi cns examination 1
Dr magdi sasi  cns  examination 1Dr magdi sasi  cns  examination 1
Dr magdi sasi cns examination 1
 
Cardiology lecture toIternal Medicine 21/10/2013
Cardiology lecture toIternal Medicine 21/10/2013Cardiology lecture toIternal Medicine 21/10/2013
Cardiology lecture toIternal Medicine 21/10/2013
 
Cases in cardiology part one PART FOUR 2016--
Cases in cardiology part one PART FOUR 2016--Cases in cardiology part one PART FOUR 2016--
Cases in cardiology part one PART FOUR 2016--
 
Cardiology presentation 2 im 762012
Cardiology presentation 2 im 762012Cardiology presentation 2 im 762012
Cardiology presentation 2 im 762012
 
Cardio respiratory nuclear imaging ihab - copy
Cardio respiratory nuclear imaging ihab - copyCardio respiratory nuclear imaging ihab - copy
Cardio respiratory nuclear imaging ihab - copy
 
ACUTE and CHRONIC AORTIC INSUFFICIENCY-DR MAGDI SASI 2016
ACUTE and CHRONIC AORTIC INSUFFICIENCY-DR MAGDI SASI 2016ACUTE and CHRONIC AORTIC INSUFFICIENCY-DR MAGDI SASI 2016
ACUTE and CHRONIC AORTIC INSUFFICIENCY-DR MAGDI SASI 2016
 
DBMS Multiple Choice Questions
DBMS Multiple Choice QuestionsDBMS Multiple Choice Questions
DBMS Multiple Choice Questions
 
rheumatic_feve for dentist 201`6--DR MAGDI SASI
rheumatic_feve for dentist 201`6--DR MAGDI SASIrheumatic_feve for dentist 201`6--DR MAGDI SASI
rheumatic_feve for dentist 201`6--DR MAGDI SASI
 
Ssc mcq-question-ans-2-motion
Ssc mcq-question-ans-2-motionSsc mcq-question-ans-2-motion
Ssc mcq-question-ans-2-motion
 

Semelhante a Mcq in cardiology 2015 magdi sasi

Anaesthetic management of a patient with mitral stenosis put for non-cardiac ...
Anaesthetic management of a patient with mitral stenosis put for non-cardiac ...Anaesthetic management of a patient with mitral stenosis put for non-cardiac ...
Anaesthetic management of a patient with mitral stenosis put for non-cardiac ...Ankur Khandelwal
 
23204910
2320491023204910
23204910radgirl
 
Management of aorto arteritis
Management of aorto arteritisManagement of aorto arteritis
Management of aorto arteritisIndia CTVS
 
03 msu disease of the vessels hajhamad m
03 msu disease of the vessels hajhamad m03 msu disease of the vessels hajhamad m
03 msu disease of the vessels hajhamad mMohammed M. H. Hajhamad
 
Constrictive pericarditis
Constrictive pericarditisConstrictive pericarditis
Constrictive pericarditishodmedicine
 
Congenital Heart Disease.pptx
Congenital Heart Disease.pptxCongenital Heart Disease.pptx
Congenital Heart Disease.pptxRashi773374
 
CAD AND MI (CORONARY ARTERY DISEASE AND MYOCARDIAL INFARCTION)
CAD AND MI (CORONARY ARTERY DISEASE AND MYOCARDIAL INFARCTION)CAD AND MI (CORONARY ARTERY DISEASE AND MYOCARDIAL INFARCTION)
CAD AND MI (CORONARY ARTERY DISEASE AND MYOCARDIAL INFARCTION)kalyan kumar
 
Complicated Tubercular Pericarditis Presenting as Ventricular Apical Aneurysm...
Complicated Tubercular Pericarditis Presenting as Ventricular Apical Aneurysm...Complicated Tubercular Pericarditis Presenting as Ventricular Apical Aneurysm...
Complicated Tubercular Pericarditis Presenting as Ventricular Apical Aneurysm...Crimsonpublisherssmoaj
 
strokeppt-170720174010.pdf
strokeppt-170720174010.pdfstrokeppt-170720174010.pdf
strokeppt-170720174010.pdfRiyaSharma295
 
ANEURYSMS , TYPES AND THERE MANAGEMENT.pptx
ANEURYSMS , TYPES  AND  THERE MANAGEMENT.pptxANEURYSMS , TYPES  AND  THERE MANAGEMENT.pptx
ANEURYSMS , TYPES AND THERE MANAGEMENT.pptxBipul Thakur
 

Semelhante a Mcq in cardiology 2015 magdi sasi (20)

Anaesthetic management of a patient with mitral stenosis put for non-cardiac ...
Anaesthetic management of a patient with mitral stenosis put for non-cardiac ...Anaesthetic management of a patient with mitral stenosis put for non-cardiac ...
Anaesthetic management of a patient with mitral stenosis put for non-cardiac ...
 
ECG in young
ECG in youngECG in young
ECG in young
 
23204910
2320491023204910
23204910
 
Management of aorto arteritis
Management of aorto arteritisManagement of aorto arteritis
Management of aorto arteritis
 
Coarctation of aorta
Coarctation of aortaCoarctation of aorta
Coarctation of aorta
 
Chronic constrictive pericarditis
Chronic constrictive pericarditisChronic constrictive pericarditis
Chronic constrictive pericarditis
 
Chronic constrictive pericarditis
Chronic constrictive pericarditisChronic constrictive pericarditis
Chronic constrictive pericarditis
 
Hocm
HocmHocm
Hocm
 
03 msu disease of the vessels hajhamad m
03 msu disease of the vessels hajhamad m03 msu disease of the vessels hajhamad m
03 msu disease of the vessels hajhamad m
 
Cardiac tamponade Toufiqur Rahman
Cardiac tamponade Toufiqur RahmanCardiac tamponade Toufiqur Rahman
Cardiac tamponade Toufiqur Rahman
 
Rvmi.+ case
Rvmi.+ caseRvmi.+ case
Rvmi.+ case
 
Constrictive pericarditis
Constrictive pericarditisConstrictive pericarditis
Constrictive pericarditis
 
Congenital Heart Disease.pptx
Congenital Heart Disease.pptxCongenital Heart Disease.pptx
Congenital Heart Disease.pptx
 
CAD AND MI (CORONARY ARTERY DISEASE AND MYOCARDIAL INFARCTION)
CAD AND MI (CORONARY ARTERY DISEASE AND MYOCARDIAL INFARCTION)CAD AND MI (CORONARY ARTERY DISEASE AND MYOCARDIAL INFARCTION)
CAD AND MI (CORONARY ARTERY DISEASE AND MYOCARDIAL INFARCTION)
 
Coarctation of aorta
Coarctation of aortaCoarctation of aorta
Coarctation of aorta
 
Complicated Tubercular Pericarditis Presenting as Ventricular Apical Aneurysm...
Complicated Tubercular Pericarditis Presenting as Ventricular Apical Aneurysm...Complicated Tubercular Pericarditis Presenting as Ventricular Apical Aneurysm...
Complicated Tubercular Pericarditis Presenting as Ventricular Apical Aneurysm...
 
strokeppt-170720174010.pdf
strokeppt-170720174010.pdfstrokeppt-170720174010.pdf
strokeppt-170720174010.pdf
 
Stroke ppt
Stroke pptStroke ppt
Stroke ppt
 
ANEURYSMS , TYPES AND THERE MANAGEMENT.pptx
ANEURYSMS , TYPES  AND  THERE MANAGEMENT.pptxANEURYSMS , TYPES  AND  THERE MANAGEMENT.pptx
ANEURYSMS , TYPES AND THERE MANAGEMENT.pptx
 
Principles of cardiology total.samir rafla
Principles of cardiology total.samir raflaPrinciples of cardiology total.samir rafla
Principles of cardiology total.samir rafla
 

Mais de cardilogy

Pud ms 2021 fifth year
Pud ms 2021 fifth yearPud ms 2021 fifth year
Pud ms 2021 fifth yearcardilogy
 
Motor function of brain and brain stem ms 2018 dentist MAGDI SASI
Motor function of brain and brain stem  ms 2018  dentist  MAGDI SASIMotor function of brain and brain stem  ms 2018  dentist  MAGDI SASI
Motor function of brain and brain stem ms 2018 dentist MAGDI SASIcardilogy
 
Bp 2021 blood flow physiological factors magdi sasi
Bp 2021 blood flow physiological factors magdi sasiBp 2021 blood flow physiological factors magdi sasi
Bp 2021 blood flow physiological factors magdi sasicardilogy
 
Role of kidney in acid base balance saturday interactive lecture m MAGDI AWAD...
Role of kidney in acid base balance saturday interactive lecture m MAGDI AWAD...Role of kidney in acid base balance saturday interactive lecture m MAGDI AWAD...
Role of kidney in acid base balance saturday interactive lecture m MAGDI AWAD...cardilogy
 
Labratory data ms 2021
Labratory data  ms 2021Labratory data  ms 2021
Labratory data ms 2021cardilogy
 
General examination ms 2020
General examination ms 2020General examination ms 2020
General examination ms 2020cardilogy
 
Chest examination magdi sasi2021
Chest examination magdi sasi2021Chest examination magdi sasi2021
Chest examination magdi sasi2021cardilogy
 
Abdomen examination ms 2021
Abdomen examination ms 2021Abdomen examination ms 2021
Abdomen examination ms 2021cardilogy
 
Heart examination magdi sasi2021
Heart examination magdi sasi2021Heart examination magdi sasi2021
Heart examination magdi sasi2021cardilogy
 
Respiration mechanics ms for dentist
Respiration mechanics  ms  for dentistRespiration mechanics  ms  for dentist
Respiration mechanics ms for dentistcardilogy
 
Regulation of respiration 2020 ms
Regulation of respiration  2020 msRegulation of respiration  2020 ms
Regulation of respiration 2020 mscardilogy
 
Cvs introduction ms 2020
Cvs introduction ms 2020Cvs introduction ms 2020
Cvs introduction ms 2020cardilogy
 
History series case one by magdi sasi 2020
History series   case one by magdi sasi 2020History series   case one by magdi sasi 2020
History series case one by magdi sasi 2020cardilogy
 
Stretch reflex imu m sasi 2020
Stretch reflex imu  m sasi 2020Stretch reflex imu  m sasi 2020
Stretch reflex imu m sasi 2020cardilogy
 
Conductivity and excitabilitry limu ms 2017.2 nd year
Conductivity and excitabilitry  limu  ms 2017.2 nd yearConductivity and excitabilitry  limu  ms 2017.2 nd year
Conductivity and excitabilitry limu ms 2017.2 nd yearcardilogy
 
Regulation of ABP magdi sasi 2018
Regulation of ABP  magdi sasi 2018Regulation of ABP  magdi sasi 2018
Regulation of ABP magdi sasi 2018cardilogy
 
Motor function of brain and brain stem ms 2017 dentist
Motor function of brain and brain stem  ms 2017  dentistMotor function of brain and brain stem  ms 2017  dentist
Motor function of brain and brain stem ms 2017 dentistcardilogy
 
Glomerular disease postgraduate magdi sasi 2019
Glomerular disease postgraduate  magdi sasi 2019Glomerular disease postgraduate  magdi sasi 2019
Glomerular disease postgraduate magdi sasi 2019cardilogy
 
Chronic myeloid leukemia magdi sasi 2019 ramadan
Chronic myeloid leukemia magdi sasi 2019 ramadanChronic myeloid leukemia magdi sasi 2019 ramadan
Chronic myeloid leukemia magdi sasi 2019 ramadancardilogy
 
Immunity introduction ms 2019 new
Immunity introduction ms 2019 newImmunity introduction ms 2019 new
Immunity introduction ms 2019 newcardilogy
 

Mais de cardilogy (20)

Pud ms 2021 fifth year
Pud ms 2021 fifth yearPud ms 2021 fifth year
Pud ms 2021 fifth year
 
Motor function of brain and brain stem ms 2018 dentist MAGDI SASI
Motor function of brain and brain stem  ms 2018  dentist  MAGDI SASIMotor function of brain and brain stem  ms 2018  dentist  MAGDI SASI
Motor function of brain and brain stem ms 2018 dentist MAGDI SASI
 
Bp 2021 blood flow physiological factors magdi sasi
Bp 2021 blood flow physiological factors magdi sasiBp 2021 blood flow physiological factors magdi sasi
Bp 2021 blood flow physiological factors magdi sasi
 
Role of kidney in acid base balance saturday interactive lecture m MAGDI AWAD...
Role of kidney in acid base balance saturday interactive lecture m MAGDI AWAD...Role of kidney in acid base balance saturday interactive lecture m MAGDI AWAD...
Role of kidney in acid base balance saturday interactive lecture m MAGDI AWAD...
 
Labratory data ms 2021
Labratory data  ms 2021Labratory data  ms 2021
Labratory data ms 2021
 
General examination ms 2020
General examination ms 2020General examination ms 2020
General examination ms 2020
 
Chest examination magdi sasi2021
Chest examination magdi sasi2021Chest examination magdi sasi2021
Chest examination magdi sasi2021
 
Abdomen examination ms 2021
Abdomen examination ms 2021Abdomen examination ms 2021
Abdomen examination ms 2021
 
Heart examination magdi sasi2021
Heart examination magdi sasi2021Heart examination magdi sasi2021
Heart examination magdi sasi2021
 
Respiration mechanics ms for dentist
Respiration mechanics  ms  for dentistRespiration mechanics  ms  for dentist
Respiration mechanics ms for dentist
 
Regulation of respiration 2020 ms
Regulation of respiration  2020 msRegulation of respiration  2020 ms
Regulation of respiration 2020 ms
 
Cvs introduction ms 2020
Cvs introduction ms 2020Cvs introduction ms 2020
Cvs introduction ms 2020
 
History series case one by magdi sasi 2020
History series   case one by magdi sasi 2020History series   case one by magdi sasi 2020
History series case one by magdi sasi 2020
 
Stretch reflex imu m sasi 2020
Stretch reflex imu  m sasi 2020Stretch reflex imu  m sasi 2020
Stretch reflex imu m sasi 2020
 
Conductivity and excitabilitry limu ms 2017.2 nd year
Conductivity and excitabilitry  limu  ms 2017.2 nd yearConductivity and excitabilitry  limu  ms 2017.2 nd year
Conductivity and excitabilitry limu ms 2017.2 nd year
 
Regulation of ABP magdi sasi 2018
Regulation of ABP  magdi sasi 2018Regulation of ABP  magdi sasi 2018
Regulation of ABP magdi sasi 2018
 
Motor function of brain and brain stem ms 2017 dentist
Motor function of brain and brain stem  ms 2017  dentistMotor function of brain and brain stem  ms 2017  dentist
Motor function of brain and brain stem ms 2017 dentist
 
Glomerular disease postgraduate magdi sasi 2019
Glomerular disease postgraduate  magdi sasi 2019Glomerular disease postgraduate  magdi sasi 2019
Glomerular disease postgraduate magdi sasi 2019
 
Chronic myeloid leukemia magdi sasi 2019 ramadan
Chronic myeloid leukemia magdi sasi 2019 ramadanChronic myeloid leukemia magdi sasi 2019 ramadan
Chronic myeloid leukemia magdi sasi 2019 ramadan
 
Immunity introduction ms 2019 new
Immunity introduction ms 2019 newImmunity introduction ms 2019 new
Immunity introduction ms 2019 new
 

Último

Call Girls Varanasi Just Call 8250077686 Top Class Call Girl Service Available
Call Girls Varanasi Just Call 8250077686 Top Class Call Girl Service AvailableCall Girls Varanasi Just Call 8250077686 Top Class Call Girl Service Available
Call Girls Varanasi Just Call 8250077686 Top Class Call Girl Service AvailableDipal Arora
 
Manyata Tech Park ( Call Girls ) Bangalore ✔ 6297143586 ✔ Hot Model With Sexy...
Manyata Tech Park ( Call Girls ) Bangalore ✔ 6297143586 ✔ Hot Model With Sexy...Manyata Tech Park ( Call Girls ) Bangalore ✔ 6297143586 ✔ Hot Model With Sexy...
Manyata Tech Park ( Call Girls ) Bangalore ✔ 6297143586 ✔ Hot Model With Sexy...vidya singh
 
(👑VVIP ISHAAN ) Russian Call Girls Service Navi Mumbai🖕9920874524🖕Independent...
(👑VVIP ISHAAN ) Russian Call Girls Service Navi Mumbai🖕9920874524🖕Independent...(👑VVIP ISHAAN ) Russian Call Girls Service Navi Mumbai🖕9920874524🖕Independent...
(👑VVIP ISHAAN ) Russian Call Girls Service Navi Mumbai🖕9920874524🖕Independent...Taniya Sharma
 
Best Rate (Patna ) Call Girls Patna ⟟ 8617370543 ⟟ High Class Call Girl In 5 ...
Best Rate (Patna ) Call Girls Patna ⟟ 8617370543 ⟟ High Class Call Girl In 5 ...Best Rate (Patna ) Call Girls Patna ⟟ 8617370543 ⟟ High Class Call Girl In 5 ...
Best Rate (Patna ) Call Girls Patna ⟟ 8617370543 ⟟ High Class Call Girl In 5 ...Dipal Arora
 
Call Girls Horamavu WhatsApp Number 7001035870 Meeting With Bangalore Escorts
Call Girls Horamavu WhatsApp Number 7001035870 Meeting With Bangalore EscortsCall Girls Horamavu WhatsApp Number 7001035870 Meeting With Bangalore Escorts
Call Girls Horamavu WhatsApp Number 7001035870 Meeting With Bangalore Escortsvidya singh
 
♛VVIP Hyderabad Call Girls Chintalkunta🖕7001035870🖕Riya Kappor Top Call Girl ...
♛VVIP Hyderabad Call Girls Chintalkunta🖕7001035870🖕Riya Kappor Top Call Girl ...♛VVIP Hyderabad Call Girls Chintalkunta🖕7001035870🖕Riya Kappor Top Call Girl ...
♛VVIP Hyderabad Call Girls Chintalkunta🖕7001035870🖕Riya Kappor Top Call Girl ...astropune
 
Call Girls Kochi Just Call 8250077686 Top Class Call Girl Service Available
Call Girls Kochi Just Call 8250077686 Top Class Call Girl Service AvailableCall Girls Kochi Just Call 8250077686 Top Class Call Girl Service Available
Call Girls Kochi Just Call 8250077686 Top Class Call Girl Service AvailableDipal Arora
 
Call Girls Haridwar Just Call 8250077686 Top Class Call Girl Service Available
Call Girls Haridwar Just Call 8250077686 Top Class Call Girl Service AvailableCall Girls Haridwar Just Call 8250077686 Top Class Call Girl Service Available
Call Girls Haridwar Just Call 8250077686 Top Class Call Girl Service AvailableDipal Arora
 
Call Girls Ooty Just Call 8250077686 Top Class Call Girl Service Available
Call Girls Ooty Just Call 8250077686 Top Class Call Girl Service AvailableCall Girls Ooty Just Call 8250077686 Top Class Call Girl Service Available
Call Girls Ooty Just Call 8250077686 Top Class Call Girl Service AvailableDipal Arora
 
Top Rated Bangalore Call Girls Mg Road ⟟ 9332606886 ⟟ Call Me For Genuine S...
Top Rated Bangalore Call Girls Mg Road ⟟   9332606886 ⟟ Call Me For Genuine S...Top Rated Bangalore Call Girls Mg Road ⟟   9332606886 ⟟ Call Me For Genuine S...
Top Rated Bangalore Call Girls Mg Road ⟟ 9332606886 ⟟ Call Me For Genuine S...narwatsonia7
 
Call Girls Bangalore Just Call 8250077686 Top Class Call Girl Service Available
Call Girls Bangalore Just Call 8250077686 Top Class Call Girl Service AvailableCall Girls Bangalore Just Call 8250077686 Top Class Call Girl Service Available
Call Girls Bangalore Just Call 8250077686 Top Class Call Girl Service AvailableDipal Arora
 
Call Girls Siliguri Just Call 8250077686 Top Class Call Girl Service Available
Call Girls Siliguri Just Call 8250077686 Top Class Call Girl Service AvailableCall Girls Siliguri Just Call 8250077686 Top Class Call Girl Service Available
Call Girls Siliguri Just Call 8250077686 Top Class Call Girl Service AvailableDipal Arora
 
Russian Escorts Girls Nehru Place ZINATHI 🔝9711199012 ☪ 24/7 Call Girls Delhi
Russian Escorts Girls  Nehru Place ZINATHI 🔝9711199012 ☪ 24/7 Call Girls DelhiRussian Escorts Girls  Nehru Place ZINATHI 🔝9711199012 ☪ 24/7 Call Girls Delhi
Russian Escorts Girls Nehru Place ZINATHI 🔝9711199012 ☪ 24/7 Call Girls DelhiAlinaDevecerski
 
Call Girls Ludhiana Just Call 9907093804 Top Class Call Girl Service Available
Call Girls Ludhiana Just Call 9907093804 Top Class Call Girl Service AvailableCall Girls Ludhiana Just Call 9907093804 Top Class Call Girl Service Available
Call Girls Ludhiana Just Call 9907093804 Top Class Call Girl Service AvailableDipal Arora
 
Call Girls Bhubaneswar Just Call 9907093804 Top Class Call Girl Service Avail...
Call Girls Bhubaneswar Just Call 9907093804 Top Class Call Girl Service Avail...Call Girls Bhubaneswar Just Call 9907093804 Top Class Call Girl Service Avail...
Call Girls Bhubaneswar Just Call 9907093804 Top Class Call Girl Service Avail...Dipal Arora
 
Premium Bangalore Call Girls Jigani Dail 6378878445 Escort Service For Hot Ma...
Premium Bangalore Call Girls Jigani Dail 6378878445 Escort Service For Hot Ma...Premium Bangalore Call Girls Jigani Dail 6378878445 Escort Service For Hot Ma...
Premium Bangalore Call Girls Jigani Dail 6378878445 Escort Service For Hot Ma...tanya dube
 
(Rocky) Jaipur Call Girl - 09521753030 Escorts Service 50% Off with Cash ON D...
(Rocky) Jaipur Call Girl - 09521753030 Escorts Service 50% Off with Cash ON D...(Rocky) Jaipur Call Girl - 09521753030 Escorts Service 50% Off with Cash ON D...
(Rocky) Jaipur Call Girl - 09521753030 Escorts Service 50% Off with Cash ON D...indiancallgirl4rent
 
Bangalore Call Girls Nelamangala Number 9332606886 Meetin With Bangalore Esc...
Bangalore Call Girls Nelamangala Number 9332606886  Meetin With Bangalore Esc...Bangalore Call Girls Nelamangala Number 9332606886  Meetin With Bangalore Esc...
Bangalore Call Girls Nelamangala Number 9332606886 Meetin With Bangalore Esc...narwatsonia7
 
Call Girls Jabalpur Just Call 8250077686 Top Class Call Girl Service Available
Call Girls Jabalpur Just Call 8250077686 Top Class Call Girl Service AvailableCall Girls Jabalpur Just Call 8250077686 Top Class Call Girl Service Available
Call Girls Jabalpur Just Call 8250077686 Top Class Call Girl Service AvailableDipal Arora
 
Call Girls Faridabad Just Call 9907093804 Top Class Call Girl Service Available
Call Girls Faridabad Just Call 9907093804 Top Class Call Girl Service AvailableCall Girls Faridabad Just Call 9907093804 Top Class Call Girl Service Available
Call Girls Faridabad Just Call 9907093804 Top Class Call Girl Service AvailableDipal Arora
 

Último (20)

Call Girls Varanasi Just Call 8250077686 Top Class Call Girl Service Available
Call Girls Varanasi Just Call 8250077686 Top Class Call Girl Service AvailableCall Girls Varanasi Just Call 8250077686 Top Class Call Girl Service Available
Call Girls Varanasi Just Call 8250077686 Top Class Call Girl Service Available
 
Manyata Tech Park ( Call Girls ) Bangalore ✔ 6297143586 ✔ Hot Model With Sexy...
Manyata Tech Park ( Call Girls ) Bangalore ✔ 6297143586 ✔ Hot Model With Sexy...Manyata Tech Park ( Call Girls ) Bangalore ✔ 6297143586 ✔ Hot Model With Sexy...
Manyata Tech Park ( Call Girls ) Bangalore ✔ 6297143586 ✔ Hot Model With Sexy...
 
(👑VVIP ISHAAN ) Russian Call Girls Service Navi Mumbai🖕9920874524🖕Independent...
(👑VVIP ISHAAN ) Russian Call Girls Service Navi Mumbai🖕9920874524🖕Independent...(👑VVIP ISHAAN ) Russian Call Girls Service Navi Mumbai🖕9920874524🖕Independent...
(👑VVIP ISHAAN ) Russian Call Girls Service Navi Mumbai🖕9920874524🖕Independent...
 
Best Rate (Patna ) Call Girls Patna ⟟ 8617370543 ⟟ High Class Call Girl In 5 ...
Best Rate (Patna ) Call Girls Patna ⟟ 8617370543 ⟟ High Class Call Girl In 5 ...Best Rate (Patna ) Call Girls Patna ⟟ 8617370543 ⟟ High Class Call Girl In 5 ...
Best Rate (Patna ) Call Girls Patna ⟟ 8617370543 ⟟ High Class Call Girl In 5 ...
 
Call Girls Horamavu WhatsApp Number 7001035870 Meeting With Bangalore Escorts
Call Girls Horamavu WhatsApp Number 7001035870 Meeting With Bangalore EscortsCall Girls Horamavu WhatsApp Number 7001035870 Meeting With Bangalore Escorts
Call Girls Horamavu WhatsApp Number 7001035870 Meeting With Bangalore Escorts
 
♛VVIP Hyderabad Call Girls Chintalkunta🖕7001035870🖕Riya Kappor Top Call Girl ...
♛VVIP Hyderabad Call Girls Chintalkunta🖕7001035870🖕Riya Kappor Top Call Girl ...♛VVIP Hyderabad Call Girls Chintalkunta🖕7001035870🖕Riya Kappor Top Call Girl ...
♛VVIP Hyderabad Call Girls Chintalkunta🖕7001035870🖕Riya Kappor Top Call Girl ...
 
Call Girls Kochi Just Call 8250077686 Top Class Call Girl Service Available
Call Girls Kochi Just Call 8250077686 Top Class Call Girl Service AvailableCall Girls Kochi Just Call 8250077686 Top Class Call Girl Service Available
Call Girls Kochi Just Call 8250077686 Top Class Call Girl Service Available
 
Call Girls Haridwar Just Call 8250077686 Top Class Call Girl Service Available
Call Girls Haridwar Just Call 8250077686 Top Class Call Girl Service AvailableCall Girls Haridwar Just Call 8250077686 Top Class Call Girl Service Available
Call Girls Haridwar Just Call 8250077686 Top Class Call Girl Service Available
 
Call Girls Ooty Just Call 8250077686 Top Class Call Girl Service Available
Call Girls Ooty Just Call 8250077686 Top Class Call Girl Service AvailableCall Girls Ooty Just Call 8250077686 Top Class Call Girl Service Available
Call Girls Ooty Just Call 8250077686 Top Class Call Girl Service Available
 
Top Rated Bangalore Call Girls Mg Road ⟟ 9332606886 ⟟ Call Me For Genuine S...
Top Rated Bangalore Call Girls Mg Road ⟟   9332606886 ⟟ Call Me For Genuine S...Top Rated Bangalore Call Girls Mg Road ⟟   9332606886 ⟟ Call Me For Genuine S...
Top Rated Bangalore Call Girls Mg Road ⟟ 9332606886 ⟟ Call Me For Genuine S...
 
Call Girls Bangalore Just Call 8250077686 Top Class Call Girl Service Available
Call Girls Bangalore Just Call 8250077686 Top Class Call Girl Service AvailableCall Girls Bangalore Just Call 8250077686 Top Class Call Girl Service Available
Call Girls Bangalore Just Call 8250077686 Top Class Call Girl Service Available
 
Call Girls Siliguri Just Call 8250077686 Top Class Call Girl Service Available
Call Girls Siliguri Just Call 8250077686 Top Class Call Girl Service AvailableCall Girls Siliguri Just Call 8250077686 Top Class Call Girl Service Available
Call Girls Siliguri Just Call 8250077686 Top Class Call Girl Service Available
 
Russian Escorts Girls Nehru Place ZINATHI 🔝9711199012 ☪ 24/7 Call Girls Delhi
Russian Escorts Girls  Nehru Place ZINATHI 🔝9711199012 ☪ 24/7 Call Girls DelhiRussian Escorts Girls  Nehru Place ZINATHI 🔝9711199012 ☪ 24/7 Call Girls Delhi
Russian Escorts Girls Nehru Place ZINATHI 🔝9711199012 ☪ 24/7 Call Girls Delhi
 
Call Girls Ludhiana Just Call 9907093804 Top Class Call Girl Service Available
Call Girls Ludhiana Just Call 9907093804 Top Class Call Girl Service AvailableCall Girls Ludhiana Just Call 9907093804 Top Class Call Girl Service Available
Call Girls Ludhiana Just Call 9907093804 Top Class Call Girl Service Available
 
Call Girls Bhubaneswar Just Call 9907093804 Top Class Call Girl Service Avail...
Call Girls Bhubaneswar Just Call 9907093804 Top Class Call Girl Service Avail...Call Girls Bhubaneswar Just Call 9907093804 Top Class Call Girl Service Avail...
Call Girls Bhubaneswar Just Call 9907093804 Top Class Call Girl Service Avail...
 
Premium Bangalore Call Girls Jigani Dail 6378878445 Escort Service For Hot Ma...
Premium Bangalore Call Girls Jigani Dail 6378878445 Escort Service For Hot Ma...Premium Bangalore Call Girls Jigani Dail 6378878445 Escort Service For Hot Ma...
Premium Bangalore Call Girls Jigani Dail 6378878445 Escort Service For Hot Ma...
 
(Rocky) Jaipur Call Girl - 09521753030 Escorts Service 50% Off with Cash ON D...
(Rocky) Jaipur Call Girl - 09521753030 Escorts Service 50% Off with Cash ON D...(Rocky) Jaipur Call Girl - 09521753030 Escorts Service 50% Off with Cash ON D...
(Rocky) Jaipur Call Girl - 09521753030 Escorts Service 50% Off with Cash ON D...
 
Bangalore Call Girls Nelamangala Number 9332606886 Meetin With Bangalore Esc...
Bangalore Call Girls Nelamangala Number 9332606886  Meetin With Bangalore Esc...Bangalore Call Girls Nelamangala Number 9332606886  Meetin With Bangalore Esc...
Bangalore Call Girls Nelamangala Number 9332606886 Meetin With Bangalore Esc...
 
Call Girls Jabalpur Just Call 8250077686 Top Class Call Girl Service Available
Call Girls Jabalpur Just Call 8250077686 Top Class Call Girl Service AvailableCall Girls Jabalpur Just Call 8250077686 Top Class Call Girl Service Available
Call Girls Jabalpur Just Call 8250077686 Top Class Call Girl Service Available
 
Call Girls Faridabad Just Call 9907093804 Top Class Call Girl Service Available
Call Girls Faridabad Just Call 9907093804 Top Class Call Girl Service AvailableCall Girls Faridabad Just Call 9907093804 Top Class Call Girl Service Available
Call Girls Faridabad Just Call 9907093804 Top Class Call Girl Service Available
 

Mcq in cardiology 2015 magdi sasi

  • 1. Cases in cardiology DR.MAGDI AWAD SASI CCU 7TH OCTOPER HOSPITAL BENGHAZI LMB—BENGHAZI UNIVERSITY
  • 2. A 55-year-old man with a history of diabetes, hypertension, hyperlipidemia, and current smoking presents with a 2-month history of progressively worsening chest pain on exertion. Which of the following diagnostic procedures is most appropriate A. Exercise ECG testing B. Adenosine myocardial perfusion scintigraphy C. Dobutamine stress echocardiogram D. PET scan E. Cardiac catheterization
  • 4. A 35 year old woman with a history of rheumatic fever in childhood had had episodes of congestive heart failure for several years. During the last of her hospital admissions, she developed massive pulmonary thrombo-embolism and died. Autopsy examination showed that her lungs were firm and brown, and microscopically had widened, edematous and fibrosed alveolar septa, aggregates of heart failure cells in the alveoli, thickened small muscular arteries, and atherosclerosis in the larger pulmonary arteries. An acquired valvular disease was found in the heart. Which one of the following was it most likely to be A. MITRAL STENOSIS. B. TRICUSPID STENOSIS. C. AORTIC STENOSIS. D. AORTIC REGURGITATION. E. MITRAL REGURGITATION
  • 5. The Correct answer is A rheumatic heart disease (isolated mitral stenosis in 25% of all cases of rheumatic h.d., mixed mitral stenosis and regurgitation in 40 % of all cases of rheumatic h.d.,( two third of patients – women( IN ABOUT 50% OF PATIENTS WITH MITRAL STENOSIS THE RHEUMATIC FEVER MAY BE UNRECOGNISED VIRAL CARDITIS - VERY RARE CAUSE OF MITRAL STENOSIS TWO THIRDS OF CASES OCCUR IN WOMEN
  • 6. Pathophysiology -the mitral valve orifice -NORMALLY ABOUT 5 CM2 (IN DIASTOLE) - MODERATE STENOSIS (APPROX 2CM2) - PATIENTS REMAIN ASYMPTOMATIC - 1CM OR LESS IN SEVERE MITRAL STENOSIS -in mitral stenosis cardiac output may be maintained by rise pressure in LA, pulmonary venous and capillaries - it results in the loss of lung compliance and the development of exertional dyspnoea -MILD STENOSIS - ATRIAL PRESSURE RISES ONLY ON EXERCISE -SEVERE STENOSIS - RAISED ATRIAL PRESSURE IS REQIURED TO MAINTAIN CO EVEN AT REST -THE RAISED ATRIAL PRESSURE - ENLARGED DIAMETERS OF LA (LA DILATATION IN 80% OS CASES) - ATRIAL FIBRILLATION (SOMETIMES THE FIRST SIGN OF CLINICAL DETERIORATION) - LOSS OF ATRIAL CONTRACTION CONTRICUTE TO DIMINISHED VENTRICULAR FILLING - RISK OF ATRIAL THROMBOSIS AND SYSTEMIC THROMBOEMBOLISM )PARTICULARY THOSE WITH AF( -SUDDEN ONSET OF ATRIAL FIBRILLATION - SUDDEN INCREASE IN PULMONARY VENOUS PRESSURE - SOMETIMES PULMONARY OEDEMA - THE LONG-TERMED MITRAL STENOSIS AND MORE GRADUAL RISE IN PRESSURE - THE INCREASED PULMONARY VASCULAR RESISTANCE, WHICH PROTECTS AGAINST PULMONARY OEDEMA - RIGHT VENTRICULAR HYPERTROPHY AND PULMONARY HYPERTENSION (RESULTS FROM : PASSIVE BACK-PRESSURE, ARTERIORAL CONSTRICTION, OBLITERATIVE CHANGES IN PULMONARY VESSELS
  • 7. SYMPTOMS )most common in the third or fourth decade of life( EXERTIONAL DYSPNOEA, NOCTURNAL DYSPNOEA, COUGH, HEMOPTYSIS LEG, ANKLE OEDEMA, ABDOMINAL SWELLING (RIGHT VENTRICULAR FAILURE( SYMPTOMS OF ACUTE PULMONARY EDEMA (AF, PREGNANCY( SYMPTOMS SECONDARY TO ARTERIAL (RARE VENOUS) EMBOLI SYMPTOMS OF DIMINISHED CO (FATIGUE, TIREDNESS( Asymptomatic mitral stenosis - the physical signs of mitral stenosis are found before symptoms develop
  • 8. SIGNS ATRIAL FIBRILLATION MITRAL FACIES AUSCULTATION LOUD FIRST HEARD SOUND (THE STENOTIC VALVE PROLONGS ATRIAL EMPTYING, LEAFLETS REMAIN OPEN AT THE ONSET OF DIASTOLE AND ARE CLOSED SUDDENLY( OPENING SNAP (THE OPENING SNAP GETS CLOSER TO THE SECOND SOUND AS THE STENOSIS BECOMES MORE SEVERE, MAY BE INAUDIBLE IF THE VALVE IS HEAVILY CALCIFIED( MID-DIASTOLIC MURMUR (THE TURBULENT FLOW CAUSES THE LOW PITCHED DIASTOLIC MURMUR, AND OFTEN A THRILL; MURMUR IS ACCENTUATED BY EXERCISE AND DURING ATRIAL SYSTOLE- PRESYSTOLIC ACCENTUATION; IT IS HEARD BEST AT THE CARDIAC APEX WITH THE PATIENTS IN THE LEFT LATERAL DECUBITUS POSITION( SIGNS OF RAISED PULMONARY CAPILLARY PRESSURE (CREPITATIONS, PULMONARY EDEMA, EFFUSIONS( SIGNS OF PULMONARY HYPERTENSION (ABNORMAL PULSATION FELT TO THE LEFT OF THE STERNUM, DUE TO EITHER TO RIGHT VENTRICULAR HYPERTROPHY OR TO FORWARD DISPLACEMENT OF THE HEART BY DILATED LA; LOUD THE PULMONARY COMPONENT OF THE SECOND HEART SOUND; TRICUSPID REGURGITATION SECONDARY TO RIGHT VENTRICULAR DILATATION CAUSES THE (SYSTOLIC MURMUR AND SYSTOLIC WAVES IN THE VENOUS PULSE(
  • 9. INVESTIGATIONS INVESTIGATIONS THE ECG : THE LEFT ATRIAL HYPERTROPHY - P MITRALE (THE BIFID P WAVES) OR ATRIAL FIBRILLATION RIGHT VENTRICULAR HYPERTROPHY (THE EARLIEST SIGN - REDUCTION IN THE SIZE OF THE USUAL QS COMPLEX IN THE LEAD V1( CXR ENLARGED LEFT ATRIUM SIGNS OF PULMONARY VENOUS HYPERTENSION (ENLARGEMENT OF THE UPPER PULMONARY VEINS, HORIZONTAL LINEAR SHADOWS IN THE COSTOPHRENIC ANGLES( ECHOCARDIOGRAPHY: THICKENED IMMOBILE CUSPS, ENLARGED SIZE OF THE LEFT ATRIUM, REDUCED VALVE AREA, REDUCED RATE OF DIASTOLIC FILLING CARDIAC CATHETERISATION: PRESSURE GRADIENT BETWEEN LA AND LV
  • 10. A 45 year old man presented with gradually increasing shortness of breath and episodes of paroxysmal nocturnal dyspnea. His blood pressure was 140/40 mm of Hg, the pulse was bounding, and a blowing diastolic murmur was heard in the right upper parasternal region. Chest X-ray showed marked widening of the mediastinum due to dilatation of the ascending aorta. The most likely diagnosis is a. Pneumococcal endocarditis of the aorta valve. b. Atherosclerotic aneurysm of the ascending aorta. c. Syphiltic aortitis. d. Mitral stenosis. e. Calcific aortic stenosis
  • 11. The correct answer is C Pathological consequences of syphilitic aortitis The aortitis is mostly inflammation in the media and adventitia, induced scarring causes an intimal wrinkling in the aortic wall Tree bark appearance, which is chx of syphillis Inflammation and fibrosis in the aorta, just above the aortic valve cusp is very problematic Can narrow the coronary arteries, producing severe ischemia Aortic valve dilates à commisures separate from each other à when the valves close, there are gaps at the periphery à aortic regurgitation, which can be severe Many cases die from this, rather than from the ruptured aneurysm
  • 12. 65year old man had transfemoral catheterization for coronary angiography which revealed the presence of severe atherosclerotic narrowing of all major coronary arteries. Within hours of the procedure, he developed severe left lower quadrant abdominal pain. which was later associated with rebound tenderness, livido reticularis (erythematous skin rash) of the abdominal skin, increasing hypertension, cold blue toes and progressively worsening renal failure. Laparotomy revealed necrosis of the descending colon. This was resected. Microscopic examination showed amorphous eosinophilic material containing slit-like spaces occluding arteries in the submucosa. Which one of the following is the most likely explanation? A. Thromboemboli secondary to myocardial infarction B. Type B dissection of the aorta C. Hypersensitivity vasculitis D. Atheroembolism E. Polyarteritis nodosa
  • 13. The correct answer is D Discussion Atheroembolic disease increasingly is being recognized as an important cause of morbidity and mortality in the developed world. The true incidence of atheroembolism is not known, but it is likely that many if not all adults older than 70 years will have evidence of atheroembolism if it is sufficiently sought. Even when the disease presents with the classic signs of digital ischemia, livedo reticularis, and eosinophilia, other causes often are sought. More often, patients present with a myriad of symptoms because of cholesterol crystal deposition in small- and medium- sized vessels. The clinical consequences range from pancreatitis to penile gangrene. It is therefore not surprising that atheroembolism has been described as a great mimicker. To make matters worse, even when a diagnosis has been accurately established no treatment has been shown to have a significant impact on the attritional morbidity and mortality associated with atheroembolism The clinical features of atheroembolism range from subclinical disease to multisystem involvement leading to multiorgan failure with an associated high mortality . The disease typically affects white men 60 years of age and older. Established precipitants of catastrophic atheroembolism are o Preceding history of angiography or any instrumentation to the aorta. o Anticoagulant therapy. o Cardiovascular surgery. o Diabetes, hypertension, and previous vascular disease. o Thrombolytic therapy (controversial (
  • 14. A 57 year old man presents with a 35 minute history of severe chest pain radiating down his left arm, associated with shortness of breath. He denies denies a prior history of chest pain. His serum troponin levels and EKG are compatible with an acute anterior wall myocardial infarction. The histological finding you would expect in the left anterior descending artery (LAD( A. Circumferential medial fibrosis B. Circumferential intimal fibrosis C. Eccentric intact atherosclerotic plaques D. Disrupted atherosclerotic plaque with non occlusive mural thrombosis E. Disrupted atherosclerotic plaque with occlusive thrombosis
  • 15. Answer E. Disrupted atherosclerotic plaque with occlusive thrombosis The rapid closure of coronary arteries caused by occlusive thrombi is the major cause of acute myocardial infarction. Disruption of coronary atherosclerotic plaques is recognized as one trigger of coronary thrombosis. Formation of thrombosis or hematoma may cause angina pectoris or an acute coronary syndrome due to occlusive thrombosis
  • 16.  A 57 year old man presents with a 35 minute history of severe chest pain radiating down his left arm, associated with shortness of breath. He denies a prior history of chest pain. His serum troponin levels and EKG are compatible with an acute anterior wall myocardial infarction. The patient actually did well initially, but collapsed outside his hospital room suddenly 6 days later. A new loud murmur indicating mitral insufficiency was heard, but he died within 10 minutes. Of the following, which is the most likely cause of death based on this history A. Pulmonary embolism originating in left ventricular mural thrombus B. Ruptured ventricular papillary muscle C. ruptured left ventricular papillary muscle D. Acute bacterial endocarditis of mitral valve E. Mitral valve prolapse
  • 17. Answer  C. ruptured left ventricular papillary muscle Ruptures can lead to papillary muscle damage with acute mitral insufficiency, which can lead to acute hemodynamic changes on the left side.
  • 18.  The most common cause of death of in- hospital patients with myocardial infarctions A. Cardiogenic shock (severe pump failure( B. Arrhyrhmia C. Cardiac free wall rupture D. Rupture ventricular aneurysm E. Constrictive pericarditis
  • 19. Answer B. Arrhyrhmia  Ventricular fibrillation is the most common form of arrhythmic death in acute myocardial infarction. The vast majority of deaths due to ventricular fibrillation occur within the first 24 h of the onset of symptoms, and of these deaths, over half occur in the first hour.  Most out-of-hospital deaths from myocardial infarction are due to ventricular fibrillation. It may occur without warning symptoms or arrhythmias.  Over the last 30 years, with careful monitoring and prompt attention to arrhythmias, the in-hospital mortality for acute myocardial infarction has been reduced from about 30 to between 10 and 15 percent.
  • 20.  A 74 year old man presents with severe chest pain to the emergency room, and is found to have a widened mediastinum on chest radiograph and a murmur of aortic insufficiency on examination. He dies on the operating table from blood loss from a ruptured but very dilated ascending aorta. At autopsy, severe aortic atherosclerosis, especially in the ascending aorta, is seen together with a longitudinal intimal ridging resembling tree bark affecting the ascending aorta. the aortic root is also dilated. Histological sections of the aortic arch confirm the atherosclerotic plaques, and also show obliterative endarteritis of the vasa vasorum. Your diagnosis: A. Kawasaki arteritis; atherosclerosis B. Atherosclerosis with secondary bacterial aortitis C. Aortic dissection., Type A D. Syphilitic aortitis with accelerated atherosclerosis E. Takayasu arteritis
  • 21.  D. Syphilitic aortitis with accelerated atherosclerosis Syphilitic aortitis causes obliterative endarteritis of the vasa vasorum. This leads to ischemia and smooth muscle atrophy of the aortic media& can cause dilation of the aortic ring leading to aortic insufficiency.
  • 22.  A 93 year old man has a syncopal episode after being well most of his life. Workup reveals aortic stenosis and you plan to operate. You expect to find: A. A calcified triscupid aortic valve without fused commisures; a normal mitral valve for age  B. A calcified tricuspid aortic valve without fused commisures; a normal mitral valve with fused, thickened chordae  C. A calcified tricuspid aortic valve with fused commisures; a normal mitral valve for age  D. A calcified bicuspid aortic valve; a normal mitral valve for age  E. A calcified bicuspid aortic valve: a calcified pulmonic valve
  • 23. Answer A. A calcified triscupid aortic valve without fused commisures; a normal mitral valve for age
  • 24.  Pulmonary hypertension is most severe in which untreated valvular disease? A. Pulmonary stenosis B. Aortic stenosis C. Aortic insufficiency D. Mitral stenosis E. Mitral insufficiency
  • 25. Answer  D. Mitral stenosis Although not usually severe, the commonest forms of pulmonary hypertension are those that occur secondary to chronic lung disease and left ventricular failure.  Thus, pulmonary hypertension and right ventricular hypertrophy (cor pulmonale) may complicate the course of severe chronic obstructive pulmonary disease (COPD).  In addition, any cause of left ventricular failure may lead to pulmonary venous congestion and hence pulmonary hypertension  Similarly, unidentified mitral stenosis may present with symptoms of severe right heart failure. Other causes include congenital heart disease, narrowing of small blood vessels in the lung, pulmonary (emboli and primary pulmonary hypertension (PPH
  • 26.  Constrictive pericarditis is most likely in A. Uremia B. Congestive heart failure C. Rheumatic fever D. Myocardial infarction E. Tuberculosis
  • 27. A body of a 57 year old woman underwent autopsy after she was run down by the Graham Ambulance Service outside the clinic. Multiple 2-4 mm vegetations were found on an otherwise normal mitral valve. the vegetations were made of fibrin on histological section, and the underlying valve was essentially normal. The history that would be most likely when you get her clinic chart A. Long history of intravenous drug abuse with recent history of fevers to 104 degree F B. Pancreatic carcinoma being treated on an experimental protocol for chemotherapy C. History of rheuthmatic fever at age nine with long term mitral stenosis and recent onset of fevers to 103 degree F D. Long history of systemic lupus erythematosis (SLE(
  • 28.  D. Long history of systemic lupus erythematosis (SLE)  Systemic lupus erythematosus (SLE) is an autoimmune disease characterized by production of autoantibodies to parts of the cell nucleus.  Cardiac complications include pericarditis, endocarditis (heart murmurs and valvular damage secondary to LIBMAN- SACKS VEGETATIONS ON THE MITRAL VALVE), and coronary artery disease (CAD). 50% of SLE patients have endothelial damage to their heart valves  Endocarditis in the form of nonbacterial vegetations (Libman Sacks nodules) vary from mild valvular thickening (common but with little or no functional problems) to large vegetations affecting the function of the valve
  • 29.  Severe back pain of acute onset in a 26 year old woman in her 35th week of gestation for her third child A. Takayasu arteritis B. Giant cell arteritis C. Aortic dissection D. Polyarteritis nodosa E. Wegener's granulomatosis
  • 30.  D. Polyarteritis nodosa Polyarteritis nodosa (PAN) is a necrotizing systemic vasculitis involving the wall of small and medium sized arteries.  The histologic aspect is defined by the presence of fibrinoid necrosis and an infiltrate rich in neutrophil polynuclears in the artery wall and rare granulomas.  Clinical manifestations are misleading, and more often the symptoms of the disease are retrospectively related to the PAN.
  • 31.  Which one of the following is not true about the development of acute rheumatic fever?  A. IT DEVELOPS DURING THE ACUTE PHASE OF A GROUP A BETA-HEMOLYTIC STREPTOCOCCAL INFECTION OF THE THROAT.  B. IT IS NOT ASSOCIATEDWITHSTREPTOCOCCAL INFECTION OF SITES OTHERTHAN THE PHARYNX.  C. IT USUALLY OCCURS DURING THE COURSE OF EPIDEMICS OF STREPTOCOCCAL THROAT INFECTIONS IN CROWDEDSETTINGS.  D. IT IS FARMORE COMMON IN UNDERDEVELOPED COUNTRIES THAN IN THE UNITEDSTATES.  E. ITS LESIONS INVOLVE THE HEART, JOINTS, SKIN AND NERVOUS SYSTEM
  • 32.  The Correct Answer is A Acute rheumatic fever is triggered by infection with specific strains of group A streptococci which possess antigens that cross-react with human connective tissue, particularly heart valve glycoprotein... The condition usually affects children or young adults, and there is a familial variation in susceptibility...its prevalance in Western Europe and North America has progressively declined to very low levels, but it remains common in parts of asia, africa and south america, where it is still the most common cause of acquired heart disease in childhood adolescence... Rheumatic fever is a systemic illness typically presenting with fever, anorexia, lethargy and joint pains... Arthritis occurs in approximately 75% of patients and other features include skin rashes, carditis and neurological features... Anti-streptococcal antibody, anti-streptolysin antibody, positive culture for group A streptococcus, recent scarlet fever... Erythema marginatum (the expanding erythematous rash) and sydenham chorea (rapid, purposeless movements) also accompany this disorder... Carditis is the most important manifestation of rheumatic fever...carditis presents as breathlessness, palpitations or chest pain...
  • 33.  Otherfeatures consist of tachycardia, cardiac enlargement and new orchanged cardiac murmurs... A soft systolic murmuris common but non-specific...a soft mid- diastolic murmuris often due to valvulitis, with nodules forming on the mitral valve leaflets... There is a pericardial friction rub which is oftenintermittent...  cardiac failure may result eitherfromimpaired function of ventricularmuscle orfrommitral oraortic incompetence and tends to occurin a 'fulminant' formof rheumatic feverthat is more common in developing countries... Electrocardiographic changes include ST orT wave changes; conduction defects sometimes occurand may cause syncope... Mitral valve becomes thickened with fused chorda tendinea... Aortic regurgitation occurs...
  • 34.  Complications of rheumatic feverinclude:  Cardiac arrhythmias  Pericarditis  Rheumatic pneumonitis  Pulmonaryembolism  Pulmonaryinfarction  Valvedeformity, andinextremecases, congestiveheart failure.
  • 35.  The following are causes of prolonged QT interval except: A. Romano Ward syndrome B. Erythromycin C. Cisapride D. Hypocalcaemia E. Hyponatraemia
  • 36.  The most common cardiac malformation is: A. Atrial septal defect B. Endocardial cushion defect C. Ventricularseptal defect D. Aortic stenosis
  • 37.  The correct answer is C  VSD most common CHD(25%), membranous type most common, large LA/RV, LV remains normal  ASD – ostiumsecundumtype most common, large RA/RV/PA ASD;  PDA premature infants, maternal rubella, large LA/LV/aorta; ESDostiumprimumASD, Down’s, gooseneckdeformity on angio fromLV outlet obstruction  PAPVR– right SVC most common, IVC (scimitarsyndro) – most common cyanotic Tetralogy of Fallot stenosis/RVH/VSD/overriding aorta, boot-shaped heart, right aortic arch in 25%, small orconcave PA;
  • 38.  Which one of the following statements is not true?  A.THE VEGETATIONS OF INFECTIVE ENDOCARDITIS OFTEN EMBOLIZE.  B.THE VEGETATIONS OF NONBACTERIAL THROMBOTIC ENDOCARDITIS OFTEN EMBOLIZE.  C.THE VEGETATIONS OF ACUTE RHEUMATIC FEVER OFTEN EMBOLIZE.  D.THE VEGETATIONS OF SYSTEMIC LUPUS ERYTHEMATOSUS (LIBMAN-SACKS DISEASE) LACK EMBOLIC PROPENSITY.  E.FUNGAL ENDOCARDITIS VEGETATIONS ARE USUALLY VERY LARGE.
  • 39.  The Correct Answer is C Small thrombi that develop on cardiac valves are termed vegetations. Vegetations may be seen in bacterial infection of the valves (bacterial endocarditis) also seen with IV drug users, systemic lupus erythematosus, and in chronic wasting states.  The four major forms of vegetative endocarditis: A. RHD (Rheumatic Heart Disease) – Notice what looks like a string of beads which are small verrucae or small vegetations, sterile deposits of fibrin tend toward a predilection for the closing margin of the valve (not the free edge). This is where the trauma is occurring as the valve closes. B. IE (Infective Endocarditis) – The damage may start on the closing margin of the valve, particularly if the valve is already damaged. Unlike RHD, they tend to be larger and bulkier. Particularly with fungal infective endocarditis as compared to bacterial, the vegetations become very large. The larger they are, the greater the risk of breaking off and embolizing. C. NBTE (non-bacterial thrombotic endocarditis) – tend to be localized and also tend to be at the closing margin. They can grow large enough to break off and embolize depending on the circumstances. D. SLE (Libman-Sacks endocarditis) – Lesions also tend to occur at the closing margin. One of the distinctive features here is that the vegetations can be on either or both sides of the valve leaflets (notice how the artist shows the leaflet partially lifted to demonstrate the underside of the leaflet.
  • 40.  Troponin T is a sensitive and specific marker for:  a. typical angina b. crescendo angina c. myocardial infarction d. arrhythmia e. Prinzmetal's angina
  • 41.  The correct answer is C Troponin T is a specific marker of myocardial ischaemia and is sensitive providing the sample was taken 16h or more after the suspected cardiac event; earlier sampling may produce false negative results. Troponin T may stay elevated for 7-10 days post MI. In Acute Coronary Syndrome, Trop T >0.1 μg/L consistent with myocardial infarction; Trop T 0.03 – 0.1 μg/L consistent with minor myocardial damage
  • 42.  Complications of myocardial infarction include all of the following except: A. rupture of left ventricle B. rupture of papillary muscle C. mural thrombi D. vasospasm E. pericarditis
  • 43.  The correct answer is D Complications of Myocardial Infarction  RECURRENT CHEST PAIN  IF POST-MI ANGINA OCCURS (20-30%), PATIENTS OFTEN GET PTCA OR CABG  RECURRENT MI (5-20%) WITHIN FIRST SIX WEEKS CARDIAC ARRHYTHMIAS AND CONDUCTION DEFECTS (BRADY- AND TACHYARRHYTHMIAS)  EXTREMELY COMMON W/ MI, MAJOR SOURCE OF MORTALITY BEFORE PATIENT REACHES HOSPITAL  Mechanisms  ANATOMIC INTERRUPTION OF PERFUSION TO STRUCTURES OF CONDUCTION PATHWAY  ACCUMULATION OF LOCAL METABOLIC FACTORS AND ABNORMAL TRANSCELLULAR ION CONCENTRATIONS DUE TO MEMBRANE LEAKS  AUTONOMIC STIMULATION  ADMINISTRATION OF POTENTIALLY ARRYTHMOGENIC DRUGS
  • 44. Types:  VENTRICULAR FIBRILLATION  DURING FIRST 48 HOURS PROBABLY RELATED TO ELECTRICAL INSTABILITY  AFTER FIRST 48 HOURS PROBABLY RELATED TO LV DYSFUNCTION  SUPRAVENTRICULAR ARRYTHMIAS  SINUS BRADYCARDIA (EXCESSIVE VAGAL STIMULATION OR SA NODAL ISCHEMIA  SINUS TACHYCARDIA (PAIN, ANXIETY, CHF, DRUGS, VOLUME DEPLETION)  ATRIAL PREMATURE BEATS AND ATRIAL FIBRILLATION (ATRIAL ISCHEMIA OR DISTENSION SECONDARY TO LV FAILURE)  CONDUCTION BLOCKS  MAY RESULT FROM ISCHEMIA OR INCREASED VAGAL TONE  RIGHT VENTRICULAR INFARCTION  1/3 OF THOSE WITH MI OF LV INFERIOR WALL WILL ALSO GET RV INFACT B/C BOTH ARE SUPPLIED BY RIGHT CORONARY ARTERY (USUALLY(  WILL GET SIGNS OF RIGHT SIDED HEART FAILURE INCLUDING JVD, PROFOUND HYPOTENSION (LV BECOMES UNDERFILLED)  MECHANICAL COMPLICATIONS, INCLUDING:  PAPILLARY MUSCLE RUPTURE  CAN BE RAPIDLY FATAL BECAUSE OF SEVERE MITRAL REGURGITATION  VENTRICULAR FREE WALL RUPTURE  MAY OCCUR WITHIN FIRST 2 WEEKS OF MI  MORE COMMON IN WOMEN AND THOSE WITH A HX OF HYPERTENSION  RESULTS IN CARDIAC TAMPONADE, PSEUDOANEURYSM (IF RUPTURE IS INCOMPLETE AND PLUGGED WITH A THROMBUS)  VENTRICULAR SEPTAL RUPTURE
  • 45. 1. VENTRICULAR SEPTAL RUPTURE DEVELOP LEFT TO RIGHT SHUNT PRECIPITATES CHF BY VOLUME OVERLOAD OF PULMONARY CAPILLARIES 1. VENTRICULAR ANEURYSM  USUALLY OCCURS WEEKS TO MONTHS AFTER MI AS WALL PROGRESSIVELY WEAKENS, BUT NOT PEFORATED BY PHAGOCYTIC CLEARANCE OF NECROTIC TISSUE  GET LOCALIZED OUTWARD BUGLE WHEN VENTRICLE CONTRACTS  RUPTURE AND TAMPONADE DO NOT DEVELOP  COMPLICATIONS INCLUDE THROMBUS FORMATION, VENTRICULAR ARRHYTHMIAS, HEART FAILURE  GET PERSISTENT ST SEGMENT ELEVATION (WEEKS AFTER Q WAVE MI 1. “PUMP” FAILURE  IMPAIRED VENTRICULAR CONTRACTILITY AND INCREASED MYOCARDIAL STIFFNESS BOTH LEAD TO SYMPTOMS OF HEART FAILURE  SIGNS AND SYMPTOMS INCLUDE DYSPNEA, PULMONARY RALES, S3  TREATMENT INCLUDES DIURESIS AND VASODILATOR THERAPY
  • 46.  A patient has persist >2mm ST elevation in V2-6 two hours following a myocardial inraction, with hypertesnion of 205/115, he has already been given morphine and aspirin , what is the next management of choice? a) IV Nitroglycerine b) Double-bolus r-PA c) iv GTN d) iv streptokinase e) subcutaneous heparin
  • 47.  The correct answer is A 1) patients with ST-segment elevation, i.e. new ST-segment elevation at the J point with the cut-off points 0.2mV in V1through V3and 0.1mV in other leads, or 1) patients without ST-segment elevation, i.e. ST-segment depression or T wave abnormalities.  Clinically established myocardial infarction may be defined by any Q wave in leads V1through V3, or Q wave 0.03s in leads I, II, aVL, aVF, V4, V5or V6.  Neither the GISSI-2/International Trials nor the Third International Study of Infarct Survival(ISIS 3)found a difference in mortality between the use of streptokinase and tissue plasminogen activator or anistreplase. Furthermore, the addition of subcutaneous heparin did not reduce mortality compared with the use of no heparin.  However, the GUSTO Trial (Global Utilisation of Streptokinase and Tissue Plasminogen Activator for occluded coronary arteries)28 employed an accelerated t-PA (tissue type plasminogen activator) regimen given over 90min29 rather than the previously conventional period of 3h. Accelerated t-PA with concomitant APTT (activated partial thromboplastin time) adjusted intravenous heparin was reported to result in 10 fewer deaths per 1000 patients treated. The risk of stroke is higher with t-PA or anistreplase than with streptokinase.
  • 48.  In the GUSTO trial, there were three additional strokes per 1000 patients treated with accelerated t-PA and heparin in comparison with streptokinase and subcutaneous heparin, but only one of these survived with a residual deficit.  In assessing the net clinical benefit, this must be taken into account with the reduced death rate in the t-PA group. Several variants of t-PA have been studied. Double-bolus r-PA (reteplase) does not offer any advantage over accelerated t-PA except for its ease of administration. Single-bolus weight- adjusted TNK-tPA (tenecteplase) is equivalent to accelerated t- PA for 30-day mortality and associated with a significantly lower rate of non-cerebral bleeds and less need for blood transfusion.  Bolus fibrinolytic therapy may facilitate more rapid treatment in and out of the hospital and reduce the risk of medication errors. The choice of fibrinolytic agent will depend on an individual assessment of risk and benefit, and also on factors such as availability and cost. For late treated patients more fibrin-specific agents may be more effective
  • 49.  Platelet aggregation is only partly inhibited by aspirin and progress has been made with the development of platelet glycoprotein IIb/IIIa inhibitors, which block the final pathway of platelet aggregation. Angiographic trials demonstrated that the combination of GP IIb/IIIa inhibitors with half-dose fibrinolytic and reduced doses of heparin, induces similar or slightly higher TIMI grade 3 flow rates when compared with full-dose fibrinolytic alone and is associated with more complete resolutionof ST-segment elevations, suggesting an improvement in tissue reperfusion. The clinical benefit and safety of these combinations has been   tested in two large trials.No reductions in 30-day mortality or intracranial haemorrhage rates but lower rates of in-hospital reinfarction were observed, however, at the cost of an increase in (mostly spontaneous) non-cerebral bleeding complications especially in elderly patients.  Therefore, the routine use of a reduced dose fibrinolytic with abciximab or other platelet glyco-protein IIb/IIIa inhibitors cannotbe recommended. Whether this combination therapy may be beneficial in specific subgroups of patients (for example those at high risk or those likely to undergo early PCI) needs to be further evaluated.  Heparin has been extensively used during and after fibrinolysis, especially with tissue plasminogen activator. Heparin does not improve immediate clot lysis but coronary patency evaluated in the hours or days following thrombolytic therapywith tissue plasminogen activator appears to be better with intravenous heparin. No difference in patency was apparent in patients treated with either subcutaneous or intravenous heparin and streptokinase.Prolonged intravenous heparin administration has not been shown to prevent reocclusion after angiographically proven successful coronary fibrinolysis. Heparin infusion after tissue plasminogen activator therapy may be discontinued after 24–48h. Close monitoring of intravenous heparin therapy is mandatory; aPTT values over 70s are associated with higher likelihood of mortality, bleeding and reinfarction. Although no randomized trials have beenperformed, there is evidence from recent trials suggesting that more frequent monitoring of aPTT and a full weight adjustment of heparinmay decrease the risk of non-cerebral bleeding complications. 
  • 50.  Familial hypertrophic cardiomyopathy is most likely to be secondary to a mutation in: a) myosin regulatory proteins b) myosin binding protein-C c) Myosin light chains d) Troponin I e) Troponin T
  • 51.  The correct answer is B Molecular genetic research has demonstrated that familial hypertrophic cardiomyopathy is caused by a mutation in one of nine genes that encode sarcomere proteins.  Sarcomeres are the contractile units within the cardiac cells; these are composed of many proteins that are organized into thin and thick filaments. These filaments slide with respect to each other during cardiac contraction. Mutations in thick filament proteins called cardiac B myosin heavy chain or cardiac myosin binding protein-C appear to account for approximately 82% of hypertrophic cardiomyopathy. Mutations in thin filament proteins cardiac troponin T and a tropomyosin account for about 13% of hypertrophic cardiomyopathy. Mutations in two other sarcomere proteins, the myosin regulatory and essential light chains are quite rare and contribute less than 5% to hypertrophic cardiomyopathy. Although these percentages are only estimates, they indicate that more disease-causing genes will be identified. A tenth gene has been identified that is a non-sarcomere protein (a subunit of protein kinase A) is associated with individuals who have both hypertrophic cardiomyopathy and Wolf-Parkinson-White syndrome.
  • 52.  A 42-year-old man with acute renal failure is confused. His serum potassium is 8.1 mEq/L . The most likely abnormal ECG finding is  a) T wave inversion b) PR interval of 300ms c) QT interval of 0.4s d) U wave e) tall tented T waves test
  • 53.  The correct answer is E  the earliest ECG evidence of hyperkalemia usually appears in the T waves The variety of changes include:  Increased amplitude and peaking of the T wave  PR interval prolongation  QRS interval prolongation Flattening of the P wave.  A plasma potassium of >6.5mmol/l should be treated urgently unless it is an artefact. Hyperkalemia may have a variety of causes: Renal failure; Excess potassium replacement therapy;Acidosis from any one of various causes (diabetic ketoacidosis, lactic acidosis, etc.); Presence of insufficient corticosteroids (Addison's disease). As in hypokalemia, there may be a poor correlation between serum potassium levels and the typical ECG changes.
  • 54.  Which of the following is most commonly associated with an increased risk for the development of torsades de pointes? a) decreased PR interval b) decreased QRS duration c) increased PR interval d) increased QT interval e) U waves
  • 55.  The correct answer is D   There is an increased risk for the development of torsades de pointes, a potentially fatal arrhythmia, when the QT interval is prolonged. Roden reviewed the drugs most commonly implicated in QT prolongation and the clinical factors that increase the risk of torsades de pointes.  Medication-induced QT prolongation was first recognized with the use of quinidine in the 1920s. Roden lists other drugs that also are implicated in prolongation of the QT interval and may cause torsades de pointes . Use of these medications, especially when a congenital QT prolongation syndrome or other clinical risk factor is present, increases the chance that torsades de pointes may develop. Because the risk of torsades de pointes is sufficiently high at typical clinical dosages of sotalol, dofetilide, and ibutilide, the review author recommends inhospital cardiac monitoring when therapy with these agents is initiated  The risk of torsades de pointes is not related linearly to the degree of QT prolongation, although any drug that prolongs the QT interval beyond 500 msec is thought to confer an elevated risk. Heart rate exerts an important effect on the risk for associated torsades de pointes, with a greater propensity for developing the arrhythmia when bradycardia is present. 
  • 56.  A 21 year old healthy female comes to the clinic with complaints of intermittent palpitations. These episodes can last from seconds to minutes and are associated with lightheadedness, diaphoresis, and occassionally chest pain. Her ECG is below:
  • 57.
  • 58. What are the ECG finding?  Normal sinus rhythm 2) Short PR interval and delta waves consistent with Wolff- Parkinson-White (WPW) syndrome  Explanation: WPW occurs when a person is born with an "accessory pathway" that can bypass the AV node and conduct action potentials straight from the atrium to the ventricle.  Normally, the AV node acts to delay the action potentials created in the sinus node before they reach the ventricles. This normal delay is represented by the PR interval (the time it takes for the sinus nodal action potential, which creates the P wave, to reach the ventricles and cause ventricular depolarization, which creates the QRS complex). An accessory pathway can conduct action potentials very fast without any delay (unlike the AV node). Thus in WPW, after the sinus node fires creating an action potential, conduction immediately goes to the ventricles through the accessory pathway without delay creating a short PR interval (PR interval < 0.12 seconds).  Since the PR interval is so short, the P wave is very close to the QRS complex and the two usually blend together making the QRS complex appear to begin with an upslope. This upslope is called a "delta wave" which is pathogneumonic for the presence of an accessory pathway indicating WPW syndome
  • 59.
  • 60. What medications should be avoided in this patient? AV nodal blocking medications  WPW occurs when a person is born with an "accessory pathway" that can bypass the AV node and conduct action potentials straight from the atrium to the ventricle.  Since the accessory pathway has a very short refractory period, it can conduct impulses from the atria to the ventricles at very fast rates (> 400 imulses per minute). If the AV nodal conduction is blocked using the AV nodal blocking medications, more conduction can occur through the accessory pathway.  In certain arrythmias like atrial fibrillation this can cause the ventricular rate to become dangerously fast.  A good pneumonic to remember the AV nodal blocking medications is "ABCD" which stands for Adenosine, Beta- blockers, Calcium channel blockers, and Digoxin.
  • 61. In your clinic she has one of these episodes and an ECG is performed
  • 62. What are the ECG #2 findings?  Narrow complex tachycardia consistent with atrial fibrillation and WPW syndrome (less likely AVNRT, although still possible.
  • 63. What medication should be given if the rhythm caused hemodynamic instability or cardiac arrest?  Procainamide Once again the AV nodal blocking medications should not be used since blocking the AV node allows more impuses to travel through the accessory pathway thus making the ventricular rate dangerously fast, often precipitating ventricular fibrillation which is fatal. Procainamide actually slows conduction through the accessory pathway while increasing the conduction through the AV node and is the treatment of choice in patients with WPW and suspected atrial fibrillation. Patients with WPW are also at an increased risk of developing AV nodal rentry tachycardias (AVNRT) which are difficult to distinguish from atrial fibrillation on ECG due to the fast ventricular rate present in both rhythms making it hard to determine if it is irregular. Adenosine is actually relatively safe to give if the rhythm is AVNRT since no conduction is occuring in the accessory pathway as occurs with the combination of WPW and atrial fibrillation (remember AVNRT occurs with dual pathways within the AV node
  • 64.  A 55 year old male with a history of diabetes mellitus type II and hypertension presents to your clinic and has the below ECG
  • 65.
  • 66. What are the ECG findings?  The ECG findings include: Normal sinus rhythm Poor R wave progression (PRWP)
  • 67. What is the differential diagnosis of this ECG finding?  The differential diagnosis for poor R wave progression (PRWP) includes :  Old anteroseptal infarct (Q waves in V1 to v3) Left ventricular hypertrophy Left anterior fascicular block Left bundle branch block Late transition or clockwise rotation of the heart (normal varient) Wolff-Parkinson-White
  • 68.  You are the house officer on call for the telemetry floor and a nurse calls you with an abnormal rhythm seen on telemetry. The patient was admitted 2 days previously for a congestive heart failure exacerbation. He denies any symptoms including chest pain, palpitations, lightheadedness, or shortness of breath. Vital signs are normal. You ask for a 12-lead ECG which is below:
  • 69.
  • 70. What are the ECG finding?  The ECG findings include: Normal sinus rhythm  Prominent U waves A "U wave" occurs just after the T wave and significantly before the following P wave. U waves can at times be mistaken for atrial flutter waves or additional P waves indicating AV block. Also, if the U wave overlaps with the T wave it may mimic a prolonged QT interval .
  • 71.
  • 72. What is the electrophysiologic mechanism responsible for the findings in the above ECG?  Prolonged depolarization of a portion of the ventricles It is though that a special subpopulation of myocardial cells exists in the mid-myocardium that has longer action potentials and longer repolarization times.  With the presence of specific stressors (hypokalemia, bradycardia, digoxin use, or LVH), amplitude of the T wave in the special subpopulation of myocardium with a longer repolarization times increases.  Since this repolarization time is significantly longer than normal myocardium (in which repolarization creates the T wave), a distinct wave after the T wave is produced which is called the U wave.
  • 73. You go back to sleep and are soon called again for another abnormal rhythm seen on the same patient which is intermittently occurring. The patient is still asymptomatic and vital signs are still normal. Another 12-lead ECG is obtained which is below
  • 74. Dx  The ECG findings include: Normal sinus rhythm Non-sustained monomorphic ventricular tachycardia with a rate of slightly less than 300.  Non-sustained ventricular tachycardia (NSVT) can occur for multiple reasons, however it is very commonly seen in patients with systolic congestive heart failure.  Short runs of NSVT are usually asymptomatic, however if the NSVT becomes sustained it can be life-threatening.  Sustained ventricular tachycardia is usually defined as VT lasing for more than 30 seconds. VT can be monomorphic (most common) or polymorphic (a.k.a. Torsades de Pointes(
  • 75. TX  Replace electrolytes and consider defibrillator placement if criteria met  NSVT alone is not life-threatening, however it predicts an increased risk of sudden cardiac death.  Correcting electrolyte abnormalities or revascularizing the treat ischemia significantly reduces the VT.  Evidence supports the implantation of automated implanted cardiac defibrillators (AICDs) in all patients with systolic congestive heart failure with an ejection fraction of < 35%.  An AICD can quickly electrically cardiovert a patient should sustained VT occur.
  • 76. 45year-old man with a strong family history of ischaemic heart disease presents with atypical chest pains. Stress echocardiography is organised. What pharmacologic agent is likely to be used produce cardiovascular stress during Stress echocardiography in this patient? a) dipyridamole b) Dobutamine c) adenosine d) arbutamine e) Atropine sulfate
  • 77.  The correct answer is A Explanation  Stress echocardiography has been developed in recent years as an effective noninvasive test for the detection and assessment of coronary artery disease. This method combines exercise with 2-dimensional echocardiography, which can assess regional and global left ventricular function during stress.  Dobutamine infusion, a pharmacologic means of producing cardiovascular stress, appears to be an excellent alternative to exercise in echocardiographic studies. Currently, it is reserved for patients who cannot exercise at a meaningful level because of advanced age, physical deconditioning, or other factors.  Dobutamine infusion is the method used most often for pharmacologic stress echocardiography. Graded dobutamine infusion--10 to 40 micrograms/kg per minute in 3-minute stages--increases myocardial oxygen demand in a fashion similar to that of staged exercise. During the dobutamine infusion, it is apparent that heart rate, contractility, and blood pressure are increased. Dobutamine has the advantage of rapid onset of action, and its effects can be reversed by giving an intravenous beta-blocker.  A synthetic catecholamine that has a relatively short half-life (about 2 minutes) , dobutamine has strong agonist activity at the beta1 receptor and mild agonist activity at the beta2 and alpha1 receptors.  Atropine sulfate can be used to increase heart rate, if necessary, and is usually administered at the peak dobutamine dose. It is usually given as a 0.5-mg bolus and in 0.25-mg increments every 60 seconds (maximum dose, 1-1.5 mg) until the desired heart rate is achieved. Dobutamine infusion is stopped after images are acquired at peak heart rate--or sooner if the patient has tachyarrhythmias.
  • 78.  A 65 year old male with end-stage COPD gets admitted to the ICU with respiratory failure. He was found to have the below ECG
  • 79.
  • 80. DX  The ECG findings include: Multifocal atrial tachycardia (three distinct P wave morphologies) Incomplete right bundle branch block Poor R wave progression PVCs Multifocal atrial tachycardia (aka MAT) is an irregularly irregular, tachycaric rhythm in which many foci in the atium chaotically fire acting as the pacemaker of the heart instead of the sinus node. The atrial rate is not as fast as in atrial flutter or atrial fibrillation, so normal AV synchrony can occur. When the rhythm has 3 distinct P wave morphologies and the heart rate is not fast, the term "wandering atrial pacemaker" or WAP is used.
  • 81. TX  The treatment of multifocal atrial tachycardia is aimed at the underlying cause.  In our case it would be to treat the COPD exacerbation and respiratory failure.  The only medication that has been used to treat MAT is verapamil with only marginal success.  No anticoagulation is needed for MAT in contrast to atrial flutter/fibrillation since the atrium are contracting well, but simply originating in different areas.
  • 82.  Hemoptysis, hematuria and recent nosebleeds in a 43 year old man: A. Takayasu arteritis B. Giant cell arteritis C. Aortic dissection D. Polyarteritis nodosa E. Wegener's granulomatosis
  • 83.  E. Wegener's granulomatosis  This condition is uncommon and occurs in young and middle aged adults, with a slight male predominance. The vessels mainly involved in this disease are the small arteries and veins. Necrotizing granulomatous lesions of the respiratory tract, focal segmental glomerulonephritis and vasculitis of other organs characterizes this condition.  Presenting features include non-specific findings such as fever, weight loss  myalgias, arthralgias, malaise and chronic rhinitis or sinusitis. Supportive otitis, mastoiditis, hearing loss and a saddle-nose defect may occur.
  • 84.  Jones criteria for the diagnosis of acute rheumatic fever include major and minor criteria. In the presence of evidence of a preceding group A streptococcal infection, which one of the following statements is correct?  a. the diagnosis requires the presence of one major criterion only.  b. the diagnosis requires the presence of at least three major criteria.  c. the diagnosis requires the presence of two major or one major and two minor criteria.  d. three minor criteria alone are sufficient for the diagnosis.  e. two major criteria in the absence of minor criteria are insufficient for the diagnosis
  • 85.  The correct answer is C For Diagnosis of Rheumatic fever, two major criteria or one major and two minor criteria must be present.  * Major Jones Criteria (manifestations) of Rheumatic fever . Carditis . Polyarthritis . Chorea . Erythema marginatum . Subcutaneous nodules  * Minor Jones criteria of Rheumatic fever: Clinical: - Previous rheumatic fever or rheumatic heart disease - arthralgia - Fever Laboratory -↑Erythrocyte sedimentation rate (ESR) -↑C-reactive protein
  • 86.  A patient develops an acute febrile illness with shivers, nonproductive cough, and pleuritic chest pain. Five days later, he presents to the emergency room after abruptly having "coughed up" nearly a cup of blood-stained sputum. Which of the following is most likely to be seen on chest x-ray? A. A cavity with a fluid level B. Blunting of diaphragmatic costal angles C. Complete opacification of one lobe with no additional findings D. Patchy consolidation centered on bronchi E. Prominent bronchi that can be followed far out into the lung fields
  • 87.  The correct answer is A. This is a classic presentation of a pulmonary abscess. Chronic courses with less severe symptoms (with intermittent improvement following short courses of antibiotics) are also sometimes seen, particularly if the diagnosis was not suspected. Chest x-ray typically shows pneumonic opacification in which a cavity, often with a fluid level, is visible. Pulmonic abscesses can be caused by anaerobes (most common, particularly if aspiration initiated the abscess), gram-negative aerobic bacilli, and Staphylococci. Therapy is based on the organisms isolated, and should be continued for at least 4 to 6 weeks. In cases that fail to resolve, the possibility of coexisting carcinoma should be considered. Choice B is the x-ray appearance of pleural effusion. Choice C is the x-ray appearance of lobar pneumonia. Choice D is the x-ray appearance of bronchopneumonia. Choice E is the x-ray appearance of bronchiectasis.
  • 88.  Which congenital heart disease is most likely to have hypertrophic and dilated bronchial arteries as a compensatory mechanism? a. ventricular septal defect b. aortic stenosis c. atrial septal defect d. persistent truncus arteriosus e. tetralogy of Fallot
  • 89.  The Correct Answer is E  In T.O.F. Roentgenogrophic examination characteristically reveals a normal-sized, boot-shaped heart (“coeur en sabot”) with prominence of the right ventricle and a concavity in the region of the underdeveloped right ventricular outflow tract and main pulmonary artery. The pulmonary vascular markings typically are diminished, and the aortic arch and knob may be on the right side; the ascending aorta usually is large. A uniform, diffuse, fine reticular pattern of vascular markings is noted in the presence of prominent collateral vessels . The degree of obstruction to pulmonary blood flow is the principal determinant of the clinical presentation. The site of obstruction is variable. Infundibular stenosis is the only major obstruction in about 50% of patients and coexists with valvular obstruction in another 20 to 25 per cent. Supravalvular and peripheral pulmonary arterial narrowing may be observed, and unilateral absence of a pulmonary artery (usually the left) is found in a small number of patients. Circulation to the abnormal lung is accomplished by bronchial and other collateral arteries.
  • 90. THANKS FOR YOUR PATIENTS ‫ونفعت‬ ‫أثمرت‬ ‫تكون‬ ‫أن‬ ‫عسى‬ ---- ‫دعواتكم‬